Community Health Final CHAP 1-25

Pataasin ang iyong marka sa homework at exams ngayon gamit ang Quizwiz!

A,B,D

17. Which programs are funded by the Community Transformation Grant program? (Select all that apply.) A)Tobacco-free living B)Physical activity C)Autism awareness D)Healthy eating E)Managing attention-deficit disorder

C) 60 days

14.The client wants to know how long Medicare will pay for skilled needs. How long can the nurse continue to provide this care before she must seek a renewal if skilled needs continue to exist? A) 30 days B) 45 days C) 60 days D) 90 days

D) Distribution of and education on the daily use of insecticide-treated bed nets

6.The community health nurse works in a low-income country. The nurse's primary concern is to decrease the incidence of malaria. Which intervention should the nurse pursue? A) Fuel alternatives to burning wood and dung B) Sanitized drinking water C) Education regarding contraception use D) Distribution of and education on the daily use of insecticide-treated bed nets

B)Use multiple methods or perspectives

6.The nurse conducts a community assessment. The community is large and the members range from affluent executives to poor migrant workers. To most accurately assess the community, the nurse should: A)Use a questionnaire written on a fifth-grade reading level B)Use multiple methods or perspectives C)Interview a subpopulation of each group represented D)Analyze census data

D) Morbidity and mortality from noninfectious diseases have increased.

6.Which most accurately describes how disease patterns have changed from 1900 to the present? A) Morbidity and mortality from infectious diseases have increased. B) Morbidity and mortality from unintentional injuries have decreased. C) Morbidity and mortality from chronic degenerative conditions have decreased. D) Morbidity and mortality from noninfectious diseases have increased.

A)Subjective data

7. The community health nurse assesses the neighborhood in which the clients live. When observing the physical environment, the nurse notes that most of the homes in the neighborhood are well kept and the children playing in the yards are wearing clean clothing. The children appear well nourished. The nurse determines that the neighborhood has little need for a well-child clinic. This assessment is based on: A)Subjective data B)Objective data C)Cognitive analysis D)Valid data

A) Private/voluntary

7. The nurse has recently been appointed to a community-based advisory board of an agency that is interested in providing fiscally sound, high-quality care for clients they decide will be their service population. Any profit margin that is acquired is reinvested in the operations of the home healthcare service. Which type of agency is this? A) Private/voluntary B) Hospital based C) Proprietary D) Official

16.Which data are tracked under epidemiologic workplace surveillance? (Select all that apply.) A) Injuries B) Illness C) Hazards D) Exposures E) Productivity

A,B,C,D A) Injuries B) Illness C) Hazards D) Exposures

13. Screening for cholesterol is an example of which prevention level? A) Primary B) Secondary C) Tertiary D) Quaternary

B) Secondary

A)Upstream

10. As an advocate for leukemia research, the nurse along with many others succeeded in urging the U.S. Congress to debate and vote on a bill that significantly increases federal funding for this research. Which system level would such an intervention address? A)Upstream B)Mainstream C)Sidestream D)Downstream

12.Pain at the end of life is complex and multifactorial, and prevalence varies according to diagnosis and other factors. A client has somatic pain. How is this type of pain typically described? A) Burning, electrical, or tingling B) Gnawing, cramping, or aching C) Aching, throbbing, or stabbing D) Electrical, cramping, throbbing

C) Aching, throbbing, or stabbing

The nurse applies the nursing process to a case in which a client has been exposed to an environmental contaminant. The nurse considers the question,"Has the exposure pathway been interrupted?" This question would be considered during which phase of the nursing process to examine the impact of the environment on human health? A) Assessment B) Planning C) Intervention D) Evaluation

D) Evaluation

The nurse visits a community along a creek where residents have begun to report upper respiratory illnesses and asthma. After investigating, the nurse learns that several miles upstream is a hazardous material disposal site. In addition, chemicals have leached through the soil into the creek water and have been carried downstream to this community. Although children are not allowed into the creek, they do often play along the bank. Which represents the route of exposure in this situation? A) Creek water B) Chemicals C) Bank of the creek D) Inhalation

D) Inhalation

Which is an example of a radiological contaminant? A) Lead B) Mercury C) Ricin D) Radon

D) Radon

A,C,D

19.Which are among the key categories of indicators of health presented in Healthy People 2020? (Select all that apply.) A) Physical activity B) Exposure to allergens C) Responsible sexual behavior D) Tobacco use E) Occupation

The nurse screens a woman who is 6 months pregnant for alcohol use. Which screening instrument should be used? A) AUDIT-C B) CAGE C) SMAST-G D) T-ACE

D) T-ACE

10.A newly hired health and safety officer works for a large manufacturing facility. One of the officer's first jobs is to conduct a complete survey of the workplace to determine what hazards are present, the location of entries and exits, the availability of emergency equipment, and potential trouble spots. Which interventions should the officer implement to accomplish this goal? A) Ergonomics B) Occupational health history C) Root cause analysis D) Workplace walk-through

D) Workplace walk-through

21.An outbreak of foodborne botulism has occurred in the community, and the nurse is on the disaster management team to address it. Which intervention should the nurse expect to implement in this situation? A)Administration of antivirals B)Administration of antibiotics C)Distribution of personal protective equipment (PPE) D)Administration of antitoxins

D)Administration of antitoxins

The nurse assesses a 19-year-old client for alcohol abuse. Which would be the most significant risk factor for alcohol abuse in this client? A)African-American race B)Female gender C)History of type 1 diabetes D)Enrollment in college

D) Enrollment in college

C) Physician visits can take 15 to 30 minutes

1.Which characterizes physicians in France? A) Physicians are evenly distributed between rural and urban areas B) About 40% of physicians are women C) Physician visits can take 15 to 30 minutes D) Physicians see about 15 clients a day

A) 72-year-old woman with well-managed diabetes who walks 2 miles every day and takes classes at the local community college

1.Which example best illustrates the World Health Organization's definition of health? A) 72-year-old woman with well-managed diabetes who walks 2 miles every day and takes classes at the local community college B) 20-year-old man with no significant history of disease who smokes, works a stressful job, and is sedentary C) 50-year-old woman with no health complaints who is alcoholic, lives alone with little social contact, and is obese D) 98-year-old man with dementia who resides in an assisted living facility

A)Inability to find a provider who accepts Medicaid

10. A family member receives Medicaid. Which is most likely to limit access of this person to healthcare? A)Inability to find a provider who accepts Medicaid B)Pre-existing conditions C)Lack of insurance D)Large insurance premiums

A)Give rewards for a lower blood sugar level and a food diary that reflects moderate dietary compliance.

10. The nurse decided to use a learning model based on Skinner with a client with diabetes to improve compliance with dietary and blood glucose assessment requirements. Which would be the best way to implement this program? A)Give rewards for a lower blood sugar level and a food diary that reflects moderate dietary compliance. B)Eliminate the reward and punish the client when she fails to comply with dietary guidelines. C)Demonstrate positive reinforcement by decreasing dietary guidelines and blood sugar parameters. D)Institute a strict timeline to accomplish goals.

C) TRICARE

10. The nurse takes care of a dependent family member whose husband is in the military. Which government funding is most likely going to be used for home healthcare services for this client? A) Medicare B) Medicaid C) TRICARE D) Veterans Administration

D) An organization that uses endowed funds or private fundraising to address the needs of individuals, families, and populations

10. Which best describes a philanthropic organization? A) Agency that is based in only one country but provides services to many other countries B) Agency that acquires resources to help others from private rather than public sources C) A group that collects data related to healthcare use across a variety of professional and service parameters D) An organization that uses endowed funds or private fundraising to address the needs of individuals, families, and populations

B)Ethnocentrism

10.During a conference after rounds, a nurse states, "They are in the United States now; you would think that they would act like us." This nurse is exhibiting: A)Cultural humility B)Ethnocentrism C)Cultural competence D)Cross-cultural nursing

B) State

10.Several levels of public health surveillance are necessary to protect the nation's health. At what level are healthcare providers and health facilities required to report certain infectious diseases? A) Local B) State C) Federal D) International

A) 12 -room unit with negative air pressure units

10.The nurse is responsible for the construction of an isolation unit during an H5N1 outbreak. The unit is designated for only those clients with a confirmed diagnosis of H5N1. Which unit would be easily adapted to client needs? A) 12 -room unit with negative air pressure units B) 15 -room unit with a portable supply of PPEs C) 8 -room unit with four code carts D) 12 -room unit with four to six air changes per hour

B) Relates human experience to nursing practice and nursing care

10.Which best describes the function of the grand nursing theory in nursing practice? A) Presents a systematic view of related statements to describe, explain, predict, or prescribe a phenomenon of interest B) Relates human experience to nursing practice and nursing care C) Assesses health patterns in 11 areas of family health D) Engages the family as a way to identify its strengths with attention to family dynamics and concerns related to health and illness

A) Living in a community with clean, fluoridated water C) Living in neighborhood with a high crime rate D) Smoking two packs of cigarettes per day

10.Which exemplify determinants of health? (Select all that apply.) A) Living in a community with clean, fluoridated water B) Having a death rate of 500 per 100,000 population C) Living in neighborhood with a high crime rate D) Smoking two packs of cigarettes per day E) Having an infant mortality rate of 95 deaths per 1,000 live births

A) Number of true positives/ Number of true positives + Number of false negatives

10.Which formula should be used to calculate sensitivity? A) Number of true positives/ Number of true positives + Number of false negatives B) Number of true negatives/ Number of true negatives + Number of false positives C) Number of conditions or events within a designated period of time/ Population at risk during the same period of time X Base multiple D) Number of deaths from a specific disease /Number of cases of the same disease X 100

D) John Snow

10.Who was the best known epidemiologist of the 19th century and used population data and personal observations to investigate the epidemic of cholera that occurred from 1848 through 1854? A) William Farr B) John Graunt C) Florence Nightingale D) John Snow

A,C

11. A client receives Medicare, has limited income, and lives in a large city. Which factors are likely to limit the client's ability to access healthcare? (Select all that apply.) A)Inability to afford supplemental insurance B)Living in an urban environment C)High out-of-pocket expenses D)Preexisting conditions E)Lack of in-network providers in her region

B,D,E

11. Select the examples of social marketing. (Select all that apply.) A)Ad campaign for a new brand of toothpaste B)Brochure placed in obstetricians' offices promoting the benefits of breastfeeding C)Infomercial on television presenting the advantages of a piece of exercise equipment D)Billboard illustrating the dangers of texting while driving E)Posters placed in college dormitories encouraging condom use

B,C,D,E

11. The nurse is conducting a 15-minute interview with a family. On the basis of the five leading principles for interviewing, appropriate actions for the nurse to take include: (Select all that apply.) A) Having family members share their family history with you, going back several generations B) Listening actively as the family members speak C) Constructing an ecomap of the family D) Asking family members to prioritize their needs and to express their expectations regarding nursing care E) Pointing out the strong relationships they enjoy in their family and explaining how they will help them cope with the health issue of concern

D) UNITED STATES

11. Which country spends the most on healthcare? A) Canada B) England C) Japan D) United States

A)Belief in the value of cooperation

11.During an ice storm, people in one neighborhood banded together and shared the home of a neighbor who still had power and heat. Other neighbors provided food, water, and bedding. In another neighborhood the neighbors did not know one another and weathered the storm alone. Both neighborhoods are part of the same community. What characteristic did the people in the first neighborhood share that made them a subculture that those in the second neighborhood lacked? A)Belief in the value of cooperation B)Religion C)Socioeconomic status D)Geography

A) Epidemiologic triad

11.In researching cases of West Nile virus in the community, the nurse explores how the interactions among people infected with this virus, mosquitoes, and the environment they share contribute to outbreaks of this disease. Which epidemiologic model is used in this case? A) Epidemiologic triad B) Wheel of causation C) Web of causation D) Natural history

B) Avoid contact with mosquitoes D) Eliminate standing water

11.Ten documented cases of West Nile virus (WNV) have occurred recently in the community. To reduce the transmission of the virus, the nurse should educate the members of the community to: (Select all that apply.) A) Avoid contact with birds B) Avoid contact with mosquitoes C) Avoid those with a confirmed diagnosis D) Eliminate standing water E) Avoid exposure to the environment during the daylight hours

C)The information will help to effectively assess the community

11.The new community health nurse introduces herself to the local minister and attends various neighborhood meetings. The nurse learns about the ethics, values, and culture of the neighborhood. The community is vocal about the need for public safety and the care of the homeless, especially in the winter. Which is the best rationale for considering all of the influences in the neighborhood affecting their health and well-being? A)The information can be used to obtain funds to create shelters for the homeless B)A neighborhood watch may be created with police support C)The information will help to effectively assess the community D)It will help the nurse create a controlled clinical environment

A,B,D,E

13. A nurse has just finished providing teaching and care to a child with type 1 diabetes. On the basis of the five dimensions of healthcare identified by the Institute of Medicine, indicators of quality care include that the child: (Select all that apply.) A)Safely self-injected insulin in the office B)Has maintained an appropriate blood glucose level consistently for 6 months C)Was seen by the nurse 2 hours after arriving for a scheduled appointment D)Received care that was identical to that provided to six other clients of various racial backgrounds E)Received instructions directly from the nurse in the office, not via a parent

B) 3

13. National Health Expenditure Accounts are a measure of expenditures on healthcare goods and services in the United States. In the most recent set of estimates, expenditures totaled what percentage of U.S. health spending? A) 2 B) 3 C) 4 D) 5

A,C,D,E

13. The nurse researches the role of nurse-managed health centers in the city. Which are common characteristics of these centers? (Select all that apply.) A)Led by advanced-practice nurses B)Serve upper-income mothers and children C)Emphasize health promotion, disease prevention, and health education D)Provide clinical practice for nursing students and faculty E)Offer specialized programs to meet the needs of specific population aggregates

A) An enhanced ability to meet his goals on the basis of his intention and behaviors

13.A 34-year-old client is recovering from a below-the-knee amputation secondary to a motor vehicle accident. The nurse meets with the client to update his care plan, and client states that he knows that he will walk again. He asks for physical therapy schedules and timelines for prosthetics on the basis of his healing process. On the basis of knowledge of the theory of reasoned action, the client's behavior reflects: A)An enhanced ability to meet his goals on the basis of his intention and behaviors B)Goal setting that is unrealistic and irrational based on the client's diagnosis and behaviors C)A depression that will complicate the healing process D)Denial of the injury and part of the healing process

A) Campylobacter

13.A client presents with severe diarrhea. The client tells the nurse that she went to a chicken roast at church the previous weekend. The nurse suspects a foodborne illness. Which cause of bacterial foodborne illness should the nurse most suspect in this case? A) Campylobacter B) Listeria monocytogenes C) Salmonella D) Escherichia coli O157:H7

D) Refer clients who have experienced neurological deficits because of the disease to a neurologist to begin rehabilitation.

13.The community health nurse addresses an outbreak of viral spinal meningitis in the community. Referring to Leavell and Clark's natural history of disease model, which example of tertiary prevention might the nurse consider in this situation? A) Initiate vaccinations for all people who have not yet been infected. B) Familiarize yourself with the signs and symptoms of the disease so that you can provide early diagnosis and treatment of it. C) Encourage any who think they might have symptoms of the disease to come to the community health clinic to be assessed as soon as possible, to prevent associated disabilities. D) Refer clients who have experienced neurological deficits because of the disease to a neurologist to begin rehabilitation.

A)Geographic information systems

13.The community health nurse explores pedestrian access to all healthcare facilities in the community. Which tool would be most helpful in performing this task? A)Geographic information systems B)Functional health pattern C)Epidemiologic model D)Developmental model

D) Animals roaming in the fair grounds

13.The nurse admits a client to the hospital with the diagnosis of Escherichia coli O157:H7. The client is dehydrated and febrile. During the history phase of the admission, the client states that he doesn't know how he could have gotten sick at the local county fair—the hamburger he bought was well done. Upon further discussion, the nurse suspects the transmission may have occurred because of: A) Use of a gas barbecue B) Improper cleaning of the cooking tools C) Use of wood instead of charcoal to cook the meat D) Animals roaming in the fair grounds

A,D

13.The nurse investigates a family's health routines and rituals as part of the overall family assessment. Which exemplify family health routines? (Select all that apply.) A) Siblings all brushing their teeth together before bed every night B) Parents taking their 9-month-old child to the health clinic when he has a fever C) Entering a 5K run/walk charity event each year as a family D) Taking a walk together around the neighborhood after dinner each evening E) The mother having a mammogram performed every year

C) Intermittent

13.The nurse provides care covered by Medicare for a client over 2 hours during the day, 3 days during the week, for 60 days. Which type of care is the nurse providing? A) Interdisciplinary B) Skilled C) Intermittent D) Part-time

C) Health service coverage

13.The nurse recently learned that more than 100 infants and children in the community have received vaccinations at the health clinic in the past month. These data are an example of which health indicator? A) Morbidity and mortality B) Risk factors C) Health service coverage D) Health system resources

B) Incidence rate

13.The nurse wishes to calculate the probability of persons in your county developing chronic obstructive pulmonary disease over the course of a year. Which measure should the nurse use? A) Incidence density B) Incidence rate C) Relative risk ratio D) Specific rate

C)Be competent in his or her own cultural heritage

13.To demonstrate true cultural sensitivity and awareness, the nurse must: A)Travel to other countries on occasion B)Become bilingual C)Be competent in his or her own cultural heritage D)Know the practices of all major cultures in one's region of the country

D)Relapse prevention model

14. A 47-year-old client and three members of his neighborhood have had their homes robbed and belongings scattered. The client contacts the police and, with their assistance, develops a neighborhood watch. The watch is organized and 24 families have been recruited to participate. The watch will be active throughout vacation months and as needed. This organization is representative of: A)Transtheoretical model B)Theory of reasoned action C)Social learning D)Relapse prevention model

A,B,C,E

14. Nurses are effective and influential shapers of healthcare policy because of their expertise in: (Select all that apply.) A)Clinical practice B)Administration C)Education D)Political science E)Research

A,B,D,E

14. The client is a single mother of two who earns minimum wage at a part-time job and never graduated from high school. Which healthcare-related outcomes is the client at increased risk for, based on her circumstances? (Select all that apply.) A) Morbidity B) Mortality C) Genetic disorder D) Difficulty accessing care E) Negative outcomes when receiving care

A)Sustainability

14. The nurse is reviewing the sources of funding for the local community health intervention program on smoking cessation. In the federal grant proposal, the nurse elaborates on all the various local organizations and private foundations that have pledged to support the program after initial funding from the grant ends. The nurse also explains the plan for ongoing fundraising efforts for the program for years to come. With these comments, the nurse is attempting to establish which essential component of funding? A)Sustainability B)Program replication C)Clinical expertise D)Accountability

B) Listeria monocytogenes

14.A client arrives at the clinic reporting diarrhea and flu-like symptoms. On questioning, the client tells the nurse about a family gathering she attended recently where she had some smoked seafood. Which foodborne illness should the nurse most suspect in this client? A) Campylobacter B) Listeria monocytogenes C) Salmonella D) Escherichia coli O157:H7

C)Diet may not meet the cultural criteria for food choices

14.A client, diagnosed with hypertension, is prescribed a no-added-salt, low-fat diet. He agrees and repeats the teaching principles. At the next visit, the nurse notices salted meat on the counter and the client shows the nurse his food diary. The diet in the past week does not reflect the physician's orders. What cultural component may have contributed to the noncompliance with the diet order? A)Diet may be too lean B)Wife cooks according to the husband's preferences C)Diet may not meet the cultural criteria for food choices D)Food tastes bland and unappetizing

D) Extreme

14.A community health nurse visits a village in Chad. The nurse has just finished assessing a 10-year-old girl who reports always being hungry. She lives in a home with her mother and father, who both work. She says she's never been to a healthcare provider before. Which level of poverty should the nurse most suspect in this girl and her family? A) Middle B) Moderate C) Relative D) Extreme

D) Takes immunosuppressants

14.Four employees at a local restaurant have positive purified protein derivative (PPD) tuberculin skin tests and have been diagnosed with latent tuberculosis (TB) infection. Which employee would be at the greatest risk for contracting active TB disease? A) Works two jobs B) Vacations in South America C) Cooks the food D) Takes immunosuppressants

A) People from South Korea, Brazil, and Nigeria all living in the same apartment complex

14.Which exemplifies cultural diversity? A) People from South Korea, Brazil, and Nigeria all living in the same apartment complex B) A town in which everyone speaks Spanish, is originally from the same region in Mexico, and prepares similar types of food C) A university where all of the professors share the same political views D) A workplace in which accountants, lawyers, marketing executives, information technologists, and maintenance workers are all employed

D) Number of fetal deaths in one year /Number of live births plus fetal deaths in the same year X1000

14.Which formula should be used to calculate the fetal mortality rate? A) Number of deaths of children <1 year of age in one year / Number of live births in the same year × 1000 B) Number of fetal deaths plus infant deaths <7 days of age in one year/ Number of live births plus fetal deaths in same year × 1000 C) Number of deaths of children <28 days of age in one year/Number of live births in the same year ×1000 D) Number of fetal deaths in one year /Number of live births plus fetal deaths in the same year X1000

B) Actions are initiated to carry out the plan

14.Which occurs in the epidemiologic process that is similar to the implementation phase of the nursing process? A) Data are gathered from reliable sources B) Actions are initiated to carry out the plan C) Actions are evaluated and report prepared D) Further research is conducted if necessary

D)Functional health pattern

14.Which type of assessment considers a configuration of behaviors that occur sequentially over time? A)Asset-based assessment B)Epidemiologic model C)Collaborative model D)Functional health pattern

B,D,E

15. A 27-year-old client comes from a background of physical and sexual abuse. Violence victimization continued with her partners until recently. She is currently in a support group for victims of abuse. The client's social background places her at risk for: (Select all that apply.) A)Increased safe sex behaviors B)Decreased safe sex behaviors C)Decreased risk for HIV/AIDS D)Increased risk for HIV/AIDS E)Increased risk of depression

B) Program replication

15. After successfully implementing and managing a smoking cessation program in an inner-city neighborhood for 5 years, the group expands the efforts to include a nearby rural community. After a challenging first year, the nurse finds that with a few modifications the program is proving successful in the rural setting, as well. Which essential component of program design has been established by success in the rural community? A)Sustainability B)Program replication C)Clinical expertise D)Accountability

A,B,C,D

15. The Department of Health and Human Services has identified several areas for which health disparities exist among various demographic groups and need to be addressed. Which are included among these areas? (Select all that apply.) A) Infant mortality B) Cancer screening C) Cardiovascular disease D) Diabetes E) End-stage renal disease

A, B,C

15. Which represent ways nurses can actively engage in policy making? (Select all that apply.) A)Testify at congressional hearings B)Disseminate information on health issues in the media C)Serve as members of governing boards D)Document client symptoms following a physical examination E)Review articles in recent nursing journals

A,C,E

15.A client is admitted to the hospital. On admission, it is noted that the client does not speak English. It will take 20 minutes for the hospital interpreter to arrive. The nurse decides to wait for the interpreter rather than relying on the client's husband. Appropriate rationales for this decision include that the interpreter is: (Select all that apply.) A)More likely to be fluent in both languages B)More likely to be familiar with the client's symptoms C)Trained in ethics and will be better at explaining cultural differences D)More likely to understand the client's needs E)More likely familiar with technical healthcare terms

B) Are likely to multitask and have more than one career in their lifetimes

15.In comparison with other generations, members of generation X: A) Respect authority, but want recognition for their individual contributions B) Are likely to multitask and have more than one career in their lifetimes C) Prefer authoritarian leadership and direct personal interaction with superiors and others D) Are goal oriented and focused people who desire guidance

A) One level teaspoon of salt and eight level teaspoons of sugar into one quart or liter of clean drinking water

15.The client reports diarrhea, fever, and abdominal cramps 24 hours following exposure to raw eggs while cooking. An efficient and effective homemade oral rehydration solution is to stir: A) One level teaspoon of salt and eight level teaspoons of sugar into one quart or liter of clean drinking water B) Eight level teaspoons of salt and one level teaspoon of sugar into one quart or liter of clean drinking water C) One level teaspoon of salt and one level teaspoon of sugar into one quart or liter of clean drinking water D) Eight level teaspoons of salt and eight level teaspoons of sugar into one quart or liter of clean drinking water

A) Increase the agency's investment in the education and training of new healthcare workers

15.The community health nurse manager works to identify how to manage the local health workforce shortage effectively. Using knowledge of the World Health Organization's (WHO) recommendations, which suggestion should the nurse manager give to the agency? A) Increase the agency's investment in the education and training of new healthcare workers B) Offer incentives to attract workers to thriving urban areas C) Attract men to the health professions D) Provide cures for common health problems to reduce demand for health services

C) Is HIV positive

15.The nurse assesses a client in the local neighborhood clinic. He exhibits signs and symptoms of tuberculosis (TB) yet has a negative tuberculin skin test result. The nurse suspects that the physician may further test the client because he: A) Visited Central America recently B) Is recovering from jet lag C) Is HIV positive D) Works in a factory setting

A,B,D

15.The nurse assesses a community undergoing change. Five years ago, a series of setbacks caused the local foundry to close. Through interviews, the nurse discovers that generations of families worked at the foundry. Research indicates that no other businesses have been able to provide work for those who no longer work at the foundry. Developmental data gathered through this research into the community's history will help the nurse: (Select all that apply.) A)Determine what resources were available before the foundry closed B)Determine what variables may have enhanced or detracted from community development C)Develop an understanding of the community's strengths and resources today D)Understand cultural changes within the community over time E)Identify where in the community health clinics are most needed

A,C,D,E

15.The nurse reviews the official document called the Scope and Standards of Home Health Nursing Practice. Using knowledge about the document providing guidelines for nurses involved in home healthcare practice, which are included in the standards of care? (Select all that apply.) A) Assessment by collecting data about home care clients B) Implementation of nurse-centered actions in collaboration with physicians C) Diagnosis through the analysis of data D) Outcome identification that helps identify nurse-sensitive measures E) Planning nurse-sensitive interventions directed at the identified outcomes

D) Characteristic or event that has been shown to increase the probability that a specific disease or illness will develop

15.Which best describes a risk factor? A) Course of a disease or condition from the onset to resolution B) Primary measurement used to describe either the occurrence or the existence of a specific state of health or illness C) Probability or likelihood that a disease or illness will occur in a group of people who presently do not have the problem D) Characteristic or event that has been shown to increase the probability that a specific disease or illness will develop

B) Number of fetal deaths plus infant deaths <7 days of age in one year/ Number of live births plus fetal deaths in the same year ×1000

15.Which formula should be used to calculate perinatal mortality rate? A) Number of deaths of children <1 year of age in one year /Number of live births in the same year × 1000 B) Number of fetal deaths plus infant deaths <7 days of age in one year/ Number of live births plus fetal deaths in the same year ×1000 C) Number of deaths of children <28 days of age in one year/Number of live births in the same year ×1000 D) Number of fetal deaths in one year/ Number of live births plus fetal deaths in the same year x1000

A,B,E

16. A 19-year-old client is diagnosed as HIV positive. Which behaviors might be indicators of noncompliance with her drug regimen and require further assessment? (Select all that apply.) A)Depression B)Indifference to self-advocacy in sexual relationships C)Goal-oriented attitude toward treatment plan D)Client's expression of concern for the health and well-being of her children E)Client's expression of concern regarding the ability to pay for her medications

A)Make the assessment process empowering to the community

16. The community is concerned about public safety and the care of their elders living at home. When the nurse assesses the community, the nurse finds there is potential to solve their problems within the community setting without outside resources. The plans to meet the community needs are accepted by the members of the community. Which action has been most successfully demonstrated in your assessment of this community? A)Make the assessment process empowering to the community B)Assess the community through interview and share findings in a timely manner C)Involve the community when there is difficulty with the assessment D)Create an environment of change within the community

C) Institute HIV prevention measures and treatment for health workers.

16. The nurse manager at a healthcare facility in a developing nation regularly loses nurses to positions in developed nations. According to the World Health Organization, which action would be appropriate for the nurse manger to take to address this problem? A) Lobby for local legislation that prevents migration of healthcare workers. B) Encourage the facility to build workforce capacity within the country. C) Institute HIV prevention measures and treatment for health workers. D) Treat exp

A,C,D

16. The nurse reviews the job descriptions of the community health workers who work with community health nurses and other members of the program team. These workers are responsible for: (Select all that apply.) A)Serving as a cultural and language bridge to members of target population B)Performing minor surgical procedures C)Providing health education and outreach to community D)Offering assistance in accessing services E)Prescribing commonly used medications

C)Reimbursement and bonuses for physicians and hospitals for adopting electronic health records

16. Which is the primary way that U.S. federal government has encouraged improved information management in the healthcare system? A)Publishing of guidelines for increased efficiency in filing client records B)Tax deductions for the installation of new telephone systems C)Reimbursement and bonuses for physicians and hospitals for adopting electronic health records D)Grants for hiring office managers trained in health information technology

C) 7 to 10 days

16.A client reports symptoms of severe abdominal cramps, bloody diarrhea, and vomiting after eating commercially packaged salad. The nurse suspects Escherichia coli O157:H7. The client asks about the average time of resolution of these symptoms. What time frame should the nurse provide? A) 3 to 5 days B) 5 to 7 days C) 7 to 10 days D) 12 to 14 days

D) Health administrator

16.A nurse in the Canadian healthcare system is concerned because a client has been denied a surgical procedure that the nurse believes is essential to the client's recovery. Who should the nurse expect will have the most power to reverse this decision? A) Nurse practitioner B) Physicians C) Private insurance company D) Health administrator

B) 1.35%

16.Over the course of a year, there were 42 cases of institution-associated pneumonia at a hospital. During that time, 3,102 clients were at risk for acquiring this infection, with 20,411 client-days. Which is the crude incidence rate of institution-associated pneumonia at this hospital? A) 0.21% B) 1.35% C) 1.52% D) 15.20%

A)Differing views of personal space

16.The nurse cares for a client in a community health clinic. The nurse is uncomfortable because the client remains very close when communicating. This discomfort makes it difficult to concentrate on the interview. When the nurse backs away, the client stops speaking and leaves. This is an example of: A)Differing views of personal space B)Treatment limitation C)Aggressive behavior D)Cultural bias

A) Social service support B) Housing assistance E) Treatment enablers

16.The nurse is responsible for the directly observed therapy (DOT) program in the community. Which represent the best methods to ensure compliance with clients' drug regimens? (Select all that apply.) A) Social service support B) Housing assistance C) Hospitalization D) Isolation until treatment complete E) Treatment enablers

A,B.D.E

16.Which are included in the five types of health risk factors that affect families? (Select all that apply.) A) Genetics B) Age C) Intelligence D) Personal health habits and lifestyle E) Environment

C) Pathological onset, presymptomatic, manifestation of clinical disease

16.Which represents the correct order of the three stages of the natural history of disease? A) Pathological onset, manifestation of clinical disease, presymptomatic B) Presymptomatic, pathological onset, manifestation of clinical disease C) Pathological onset, presymptomatic, manifestation of clinical disease D) Manifestation of clinical disease, presymptomatic, pathological onset

C)Inability to advocate for herself

17. A 32-year-old African-American client is diagnosed with HIV. She is at her 6-month appointment following diagnosis and treatment. The client states that she has not revealed her diagnosis to her boyfriend because she is afraid that he will leave. She expresses feelings of anger because she must put her feelings and illness aside or be abandoned. The client's behaviors reflect: A)Denial of her disease process B)Noncompliance with her drug regimen C)Inability to advocate for herself D)Fear of financial insolvency

B)Decision of importance is typically made by the family as a group.

17. A new client, who is Latin American, is getting settled in the hospital. She is expecting her family to arrive soon. A physician enters and informs the client that she requires surgery and asks that she sign the necessary documents. The client refuses and becomes agitated. Which is the most likely cultural reason for the client's reluctance to sign the documents? A)Client is reluctant to have the surgery. B)Decision of importance is typically made by the family as a group. C)Client does not feel comfortable with the surgeon. D)Client is medicated and is having difficulty making a decision.

A) Clinical model

17. In a healthcare facility, a client is regarded as being healthy if he or she has no diagnosed diseases and is free of any symptoms. Which model of health is in place at this facility? A) Clinical model B) Role performance model C) Adaptive model D) Eudaimonistic model

A,B,C

17. The nurse reviews the logic model and formal evaluation plan of a grant proposal on a community rabies program. Which requirements should be included in the plan to support accountability? (Select all that apply.) A)Regular communication about funds that were not used B)Details of program activities C)Report of areas where there is no progress toward program goals D)Proof that the program can be reproduced effectively in other settings E)Explanation of how the program will be funded after grant funding expires

D)Shift the focus from dependency to empowerment

17. When the community collaborates in a plan to create services and education to enhance their health and well-being, the plan is most likely to: A)Create an environment that fosters change B)Have an increased rate of success C)Empower the community but diminish the rate of success D)Shift the focus from dependency to empowerment

A) Concentration of infectious droplet nuclei in the air B) Duration of exposure to the infected person C) Proximity to the infected person

17.As the occupational health nurse in a factory finds that five employees out of 30 have positive tuberculin skin test results. One employee on the 3 -to -11 shift has been diagnosed with active tuberculosis (TB). She asks if this means that the whole factory has been exposed. The nurse replies that the probability of infection depends upon: (Select all that apply.) A) Concentration of infectious droplet nuclei in the air B) Duration of exposure to the infected person C) Proximity to the infected person D) The use of safety gloves by employees E) Physical exertion required in the job

A) 2.1

17.Over the course of a year, there were 42 cases of institution-associated pneumonia at a hospital. During that time, 3,102 clients were at risk for acquiring this infection, covering a total of 20,411 client-days. Which is the client density in terms of number of cases per 1,000 client-days at this hospital during this time? A) 2.1 B) 13.5 C) 15.2 D) 152

A) The names of all people who have been infected by the virus B) The geographical area within which the outbreak has occurred D) The time of onset and duration of the outbreak

17.The nurse has been requested to investigate the outbreak of H1N1 in the community. In documenting the outbreak, which should be included in its description? Select all that apply. A) The names of all people who have been infected by the virus B) The geographical area within which the outbreak has occurred C) A brief history of all of the other viral outbreaks that have occurred in the community in recent years D) The time of onset and duration of the outbreak E) The anticipated economic impact of the outbreak on the community

A,B,C,E

17.The nurse is selecting a health appraisal related to alcoholism to assess health risk in a client. Which are examples of information that should be included in the appraisal? (Select all that apply.) A) Signs and symptoms of alcoholism B) Recommended daily limit of alcoholic drinks consumed per day C) Examples, such as frequently drinking an alcoholic beverage in the morning D) Incidence rates of alcoholism in the United States E) Information on local substance abuse programs

A,B,C

17.Using knowledge of the parish or faith community nurse, which interventions are likely to perform on the basis of the key roles of this type of nurse? (Select all that apply.) A) Teaching a class to faith community members on managing high blood pressure B) Providing counsel on preventing upper respiratory illness to member C) Referring a faith community member to a nutritionist for dietary counsel D) Assisting an obstetrician in the birth of a faith community member's child E) Administering vaccinations at a local health clinic

C) Two or more people having experienced a similar illness after exposure to water

17.Which criteria must be met to define a water-associated disease outbreak? A) Two or more people with laboratory-confirmed primary amebic meningoencephalitis B) Two or more cases of chemical poisoning if water quality data indicate contamination by the chemical C) Two or more people having experienced a similar illness after exposure to water D) Epidemiologic evidence implicating drinking water as the probable source of the illness

C) An average hospital stay of 9 days

17.Which is characteristic of the German healthcare system? A) Health insurance payments based on risk, not income B) Ambulatory care provided by centralized nonprofit hospitals C) An average hospital stay of 9 days D) Nurse practitioners operating independently

B) Role performance model

18. A client reluctantly came to the clinic only after his daughter insisted that he do so. The nurse finds that he has type 2 diabetes. When the nurse explains that he has a serious condition, the client brushes it off and says, ìI can still work and provide for my family. I'm not sick.î Which model of health has this client adopted? A) Clinical model B) Role performance model C) Adaptive model D) Eudaimonistic model

B)"You will not be protected against other strains of HIV or sexually transmitted infections (STIs)."

18. A woman arrives at the clinic for a routine appointment. She has been HIV positive for 3 years and states she is doing well. She currently is involved in a relationship with a man who is HIV positive. She states that she is not consistent with safe sex practices due to her significant other's preferences and feels uncomfortable asking him to wear a condom. Which is the nurse's best response to promote the client's health? A)"You won't become pregnant as long as you are taking your drug cocktail." B)"You will not be protected against other strains of HIV or sexually transmitted infections (STIs)." C)"You will both be safe because you both already have HIV." D)"Let's have you both tested for STIs."

D)Gather and analyze relevant information

18. The nurse is working through an ethical dilemma using a decision-making process developed by the Ohio Nurses Association. After identifying the existence of the ethical dilemma, the next step in this process is to: A)Clarify personal values and moral position B)Determine options on the basis of consideration of benefits and risks C)Make a responsible decision about actions or recommendations D)Gather and analyze relevant information

B)Ability to reproduce a successful program within a different community C)Ability to re

18. Which best defines program replication? A)Ability to replicate a successful program in the same community a second time B)Ability to reproduce a successful program within a different community C)Ability to replicate a successful program with a new sample aggregate D)Ability to implement a successful program with the same population aggregate

B) Dual-level payment system

18. Which type of payment system does the Netherlands have? A) Universal long-term payment system B) Dual-level payment system C) Private health insurance linked to employment D) Free health insurance paid by the government

C) Chlamydia D) Gonorrhea E) Syphilis

18.A 16-year-old client visits the community health clinic with concerns that she may have a sexually transmitted infection (STI). She asks whether STIs are treatable. STIs that are easily treated and curable include: (Select all that apply.) A) Human papillomavirus B) Herpes simplex C) Chlamydia D) Gonorrhea E) Syphilis

C) 6 -mm induration

18.A man who lives in a homeless shelter is diagnosed with active tuberculosis (TB). The shelter keeps a list of those who are assigned beds, and the majority of those sleeping there are regulars. Which finding would indicate a positive tuberculin skin test result using a two -step process? A) 4 -mm induration B) 2 -mm induration C) 6 -mm induration D) 3 -mm induration

B) Wheel of causation

18.In researching an outbreak of malaria in the developing nation where the nurse currently works, the nurse decides to not focus on the role of mosquitoes in transmitting the disease but on how the physical environment of the community, biological aspects of the community, and social customs interact to affect the prevalence of this disease. Which epidemiological model is the nurse using? A) Epidemiologic triad B) Wheel of causation C) Web of causation D) Natural history

A) 80.8%

18.In reviewing a study that considered the sensitivity of a particular screening test for HIV, the nurse found 21 true positives, 853 true negatives, 3 false positives, and 5 false negatives. What is the sensitivity of this test? A) 80.8% B) 87.5% C) 93.7% D) 99.6%

D)Collaborate with other health care workers and the clients

18.The community health nurse works in a community recently devastated by a tornado. Many of the survivors are cared for in their homes. Care is needed for those who are ill, recovering, or in hospice. To provide the most holistic and effective care to the community, the nurse should: A)Refer the clients to an outreach center B)Consult with the hospice nurse and the social worker C)Provide care according to the physician's orders D)Collaborate with other health care workers and the clients

B) 24 hours

18.The home care agency has just received a client referral from a physician and assigned the case. Within how many hours from receiving the referral should the nurse plan to make the initial visit to this client, on the basis of typical agency policy? A) 12 hours B) 24 hours C) 36 hours D) 48 hours

A,B,C,D

18.The nurse assesses a family in which the wife and mother has recently been diagnosed with breast cancer. Which factors will likely affect the family's response to the health risk that this diagnosis presents? (Select all that apply.) A) What the husband and wife learned about dealing with serious illness in their families of origin B) Whether the family is covered by health insurance C) How the children tend to cope with stressful situations D) The response of fellow members of the family's mosque E) The incidence rate of breast cancer in the family's community

A)Fatalistic behavior

18.The physician approaches the client with a treatment plan for his cancer. The client states that his life is in God's hands and refuses conventional treatment. This is an example of: A)Fatalistic behavior B)Denial C)Anger D)Bargaining

A,C,E

19. Secondary HIV prevention or positive prevention is used when caring for African-American women living with HIV by addressing which behavioral risks? (Select all that apply.) A)Interpersonal mistrust B)Alcoholism C)Poor adherence to treatment D)Smoking E)Compromised self-advocacy in sexual relationships

A,C,D

19. Select the nursing ethical principles established by the American Nurses Association's (ANA) Code of Ethics for Nurses? (Select all that apply.) A)Compassion and respect for the dignity, worth, and uniqueness of every individual B)Primary commitment to the physician C)Promotion and advocacy to protect the client's health, safety, and rights D)Participation in establishing, maintaining, and improving healthcare environments and conditions E)Self-sufficiency and lack of dependence on other healthcare professionals in caring for the client

A)Obtaining a list of the herbs and folk medications that are taken and notifying the physician

19. The client is admitted to the hospital for new-onset hypertension and chest pain. During the admission interview you ask the client if she takes over-the-counter medications, and she states no. She adds that she routinely visits a folk healer. The nurse's response should include: A)Obtaining a list of the herbs and folk medications that are taken and notifying the physician B)Requesting the client to stop the ingestion of the folk herbs C)Calling poison control with the list of herbs and folk medications D)Encouraging the client to continue to take the herbs and folk medications

C,D,E

19. The nurse is in the process of forming a coalition to support a community flu shot program. Which step should the nurse take as part of this process? (Select all that apply.) A)Assign members of the coalition to tasks in a random manner. B)Seek out people with similar opinions and roles in the community. C)Consider how each member of the coalition and the organization or group represented will benefit from the work of the coalition. D)Understand group dynamics and members' motives for serving. E)Form a group that is heterogeneous but able to show mutual respect.

C) Receives funding from both governmental and nongovernmental sources

19. Which best describes a multilateral agency? A) Is based in only one country but provides services to many other countries B) Is responsible for the health and welfare of citizens nationally C) Receives funding from both governmental and nongovernmental sources D) Acquires resources to help others from private rather than public sources

C) 29.34

19.City A has a population of people who are exposed to HIV, whereas City B has a population of people who are considered nonexposed to HIV. There were 3,267 new HIV cases in a given year for City A, with a population (exposed) of 10,000,000. There were only 5 new HIV cases in the same year for City B, with a population (nonexposed) of 150,000. Which is the attributable risk of HIV occurring in City A in terms of cases per 100,000 population? A) 3.33 B) 13.55 C) 29.34 D) 32.67

B) Tuberculosis guidelines require testing of an entire healthcare setting.

19.Each employee in a small community hospital is informed that he or she needs a tuberculin skin test and of the date that the test is due. The kitchen employees tell the nurse that they don't need the test and never had to have it before. The nurse should respond with: A) The test does not need to be given if you do not have client contact. B) Tuberculosis guidelines require testing of an entire healthcare setting. C) The test will not hurt; we won't fire you if you test positive. D) The tuberculosis guidelines state kitchen workers are at risk.

A) High fertility and high mortality, resulting in slow population growth

19.The nurse works to identify epidemiologic transitions in the community. Which is an example of a demographic transition? A) High fertility and high mortality, resulting in slow population growth B) Progressive declines in mortality, as epidemics become less frequent C) High and fluctuating mortality, due to poor health, epidemics, and famine D) Further declines in mortality, increasing life expectancy, and predominance of noncommunicable diseases

A) Having multiple sex partners D) Having a sexual partner who has had an STI

19.Which factors place an individual at high risk for acquiring a sexually transmitted infection (STI)? (Select all that apply.) A) Having multiple sexual partners B) Using a condom during sex C) Having no other STIs D) Having a sexual partner who has had an STI E) Receiving a blood transfusion

A) Bill and Melinda Gates Foundation

2. A nurse is interested in exploring initiatives related to health, poverty, and development in Africa. Which organization would most likely be involved in such an initiative? A) Bill and Melinda Gates Foundation B) Living Proof Project C) International Committee of the Red Cross D) Oxfam International

B) Health as Expanding Consciousness

2. During a home visit of a client with high blood pressure with whom the nurse has developed a strong therapeutic relationship, the nurse learn that the client keeps a large salt shaker near the stove and heavily salts whatever he is cooking. He also keeps a salt shaker on the dinner table and adds more salt when eating. The nurse suggests removing the salt shaker near the stove and only lightly salting food when eating as a way to help reduce the client's sodium intake. This intervention would most likely achieve the goals for assessment associated with which nursing theory? A) Science of Unitary Beings B) Health as Expanding Consciousness C) Roy Adaptation Model D) Self-care Agency

A)Initiation of neighborhood clinics at low or no cost to the community

2. The nurse conducts a community assessment in a town once supported by multiple paper factories. The factories closed 2 years ago and most of the population is now out of work. The school nurse notes that 68% of the students are behind schedule in their physicals and immunizations. On the basis of this community assessment, the nurse should recommend: A)Initiation of neighborhood clinics at low or no cost to the community B)Creation of a task force to assess the impact of low income on immunization compliance C)A formal petition to the local government to mandate immunizations by imposing monetary fines on parents whose children are not immunized D)A tax relief program for new companies wishing to settle in the community

B)Develop a plan to meet the costs and promote the benefits of the project

2. The nurse identifies the specific indicators and focal areas needed to establish immunization and well-child clinics in a low-income neighborhood. What actions are needed to meet the health outcomes of the project? A)Identify the need in the community for the clinic on the basis of subjective data. B)Develop a plan to meet the costs and promote the benefits of the project. C)Consider the identified neighborhood's need for healthcare compared with that of other low-income neighborhoods by interviewing the members of the neighborhood. D)Assume that the neighborhood will welcome the institution of a well-child and immunization clinic.

D) Care management

2.A community care nurse at a healthcare agency assists in coordinating a plan for providing health services in the most cost-effective way possible to a large number of members living in the region. This work is known as: A) Case management B) Home healthcare C) Interdisciplinary collaboration D) Care management

B) Portal of exit from the infected person C) Means of transmission D) Portal of entry to a susceptible host

2.A student nurse asks if an infectious disease is the same thing as a communicable disease. The nurse explains that an infectious disease is not necessarily a communicable disease. Which must a communicable disease have that an infectious disease does not have to have? (Select all that apply.) A) Pathogenic microorganism B) Portal of exit from the infected person C) Means of transmission D) Portal of entry to a susceptible host E) Organism that harbors the infectious agent

B) Develop improved defenses against reinfection

2.Nursing students in an enclosed classroom begin to become ill with a gastrointestinal illness. The public health department Page 63 identifies this occurrence as an emerging epidemic and isolates the population that is ill. According to microbial adaptation, those who survive the illness will then do which of the following? A) Recover with adaptive organism reintroduction into the environment B) Develop improved defenses against reinfection C) Become hosts to the organism, which is symbiotic in nature D) Become carriers of the infection, ultimately leading to an epidemic

B) Average number of years that a person can expect to live in ìfull healthî by taking into account years lived in less than full health due to disease and/or injury

2.The nurse collects specific data related to risk factors, health indicators, and health outcomes that determine the burden of disease. Which data would the measure known as HALE yield about health indicators and outcomes? A) Average number of years a newborn is expected to live if current mortality rates continue to apply B) Average number of years that a person can expect to live in ìfull healthî by taking into account years lived in less than full health due to disease and/or injury C) Quantifying the burden of disease to a healthy life on the basis of the loss of years related to the burden of disease. The gap in years reflects the current state of health compared with an optimum state of health of a nation. D) Quantifying the burden of acute illness to a healthy life on the basis of the loss of years related to the burden of acute illness. The gap in years reflects the current state of health compared with an optimum state of health of a nation.

B)Cultural belief

2.The nurse is caring for a client who refuses the food as served. He states that the food is foreign to him and will make him ill. All food must be blessed. The personal care attendant is upset by this behavior and states that the client should eat the food anyway. The client's behavior is most likely a result of: A)Psychosocial deficit B)Cultural belief C)Allergies D)Hygiene

C) Number of conditions or events within a designated period of time/ Population at risk during the same period of time × Base multiple

2.Which formula should be used to calculate a rate? A) Number of true positives/ Number of true positives + Number of false negatives B) Number of true negatives/ Number of true negatives + Number of false positives C) Number of conditions or events within a designated period of time/ Population at risk during the same period of time × Base multiple D) Number of deaths from a specific disease/ Number of cases of the same disease X 100

C)Posttraumatic stress disorder

20. A 43-year-old African-American client is HIV positive. She has a personal history of abuse, which ended when her husband died 2 years ago. When counseling the client about adherence to her treatment plan, the nurse finds her intelligent and goal oriented. In addition to her need for education and follow-up regarding her HIV diagnosis and treatment, which associated condition should the nurse most consider when caring for this client? A)Cardiac tamponade B)Chronic obstructive pulmonary disease C)Posttraumatic stress disorder D)Cirrhosis of the liver

C) Women's health

20. The International Council of Nurses (ICN) is a federation of more than 130 national nurses associations, representing more than 13 million nurses worldwide. In which area is the International Council of Nurses particularly active? A) Basic nursing practice B) Secondary healthcare C) Women's health D) Safe immunization programs

B)Father being challenged to stop smoking because his daughter learned of the hazards of smoking from a smoke cessation program at her middle school

20. Which best illustrates systems theory? A)School systems in two different counties each setting up their own independent afterschool exercise programs B)Father being challenged to stop smoking because his daughter learned of the hazards of smoking from a smoke cessation program at her middle school C)Hospital requiring all of its nurses to wear an identification badge while on duty D)Physician in one state deciding to stop accepting Medicaid clients, while another physician in another state begins accepting Medicaid clients

B,D,E

20. Which goals are part of the Patient Protection and Affordable Care Act (ACA), which was passed in 2010? (Select all that apply.) A)Shifting the focus of the healthcare delivery system from prevention to treatment B)Reducing the costs and improving the efficiency of healthcare C)Allowing insurers to deny healthcare coverage on the basis of preexisting conditions D)Expanding health insurance coverage E)Expanding healthcare access to children

D) A conversation the client has with his healthcare provider

20.A client is fearful of being infected with the Ebola virus. To which reliable source of family risk reduction information should this client turn? A) A report from the client's neighbor whose daughter was recently infected with the Ebola virus B) Information the client's wife reported from a newspaper article about the Ebola virus that she read C) Data from an Internet site that the client's daughter conveyed D) A conversation the client has with his healthcare provider

A) Abstinence

20.A client wants to know the best way to avoid sexually transmitted infections (STIs). The nurse should mention: A) Abstinence B) Monogamous relationship with an infected partner C) Washing the genitals after sex D) Use of latex condoms

A) Genetic and biological B) Physical/environmental D) Ecological E) Social, political, and economic

20.A nurse is studying the Institute of Medicine's convergence model to learn how four types of interrelated and overlapping microbe -human interactions can lead to emerging/reemerging infectious diseases. These four types of interactions are: (Select all that apply.) A) Genetic and biological B) Physical/environmental C) Spiritual and psychological D) Ecological E) Social, political, and economic

D) Living room and bedroom

20.The home health nurse has to ensure that a safety plan is in place to address falls. Using knowledge that 55% of fall-related injuries occur with falls inside the home, a total of more than 60% of these falls occur in which two rooms? A) Living room and hallway B) Kitchen and bathroom C) Hallway and bedroom D) Living room and bedroom

C)Nurses can facilitate social change.

20.The nurse educator is responsible for conducting educational seminars for the staff. After the education component of the program, the group engages in discussion. It is important that nurses routinely attend educational programs related to cultural competence because: A)The information is lost when caring for clients in real life. B)One can't be competent at all times. C)Nurses can facilitate social change. D)Classes are necessary for regulatory compliance. Ans:

A) Epidemic curve

20.The nurse graphs the distribution of influenza cases by the time of onset of influenza. Which would be the result of this work? A) Epidemic curve B) Epidemiologic descriptive study C) Incidence density D) Incidence rate

B)Internet—aggregate data

20.The nurse is approached by the health department to investigate and assess the rise in developmental disabilities in an isolated section in the community. To begin the assessment, the nurse should gather objective data from which source? A)Client interview—aggregate data B)Internet—aggregate data C)Internet—Wikipedia D)Direct observation of the area

A) Returns to the home country with advanced practice skills B) Returns to the home country and teach others what was learned

20.The nurse is migrating from a developing nation to a developed nation. This migration ultimately benefits the nurse's home country if the nurse: (Select all that apply.) A) Returns to the home country with advanced practice skills B) Returns to the home country and teach others what was learned C) Shares with the new coworkers nursing practices unique to the nurse's home country D) Earn more money in the position in the new country E) Is safer from exposure to HIV in the new country

C) Florence Nightingale

20.Who used statistics to improve public health in England? A) William Farr B) John Graunt C) Florence Nightingale D) John Snow

B)Selective screening

21. The nurse works at community clinic that provides various types of health screenings for members of the community. One day, the nurse provides tuberculin tests for a group of new hospital employees. Which type of screening procedure is this? A)Mass screening B)Selective screening C)Multiphasic screening D)Case finding

A) Expressing empathy

21. The nurse works with 68-year-old client who is having great difficulty managing his blood glucose level. The nurse finds that the client is not following the diet that he had agreed to follow and is eating portions that are far too large. He apologizes and explains that he is grieving the loss of his wife, who died 2 weeks ago. The nurse shares condolences with him on his loss and explain that it is perfectly understandable to have trouble following a new diet while grieving. In this scenario, the nurse is using which principal approach in motivational interviewing? A) Expressing empathy B) Supporting self-efficacy C) Rolling with resistance D) Finding a discrepancy

A,E

21. Which steps must take place to improve the U.S. healthcare system, according to Porter? (Select all that apply.) A)Measurement and dissemination of health outcomes must be shared. B)The existing system of delivery of prevention, wellness, screening, and routine health maintenance services must be preserved. C)Care delivery must be reorganized around the availability of physicians. D)Providers must compete for nurses on the basis of the quality of the care provided. E)Clients' involvement in their health must improve and incentives for client involvement should be considered.

C) Syphilis

21.A client presents with several ulcerative sores on his penis that are firm, round, and painless. He also has rough, reddish brown spots on his palms and the bottoms of his feet. He acknowledges that he is sexually active with multiple partners and is inconsistent in his use of a condom. Which sexually transmitted infection should the nurse most suspect in this client? A) Gonorrhea B) Chlamydia C) Syphilis D) Human papillomavirus

B) Providing sanitized drinking water

21.The community health nurse in a low-income country is working to decrease the incidence of diarrheal illnesses. Which intervention would most likely best address this problem? A) Providing fuel alternatives to burning wood and dung B) Providing sanitized drinking water C) Education regarding contraception use D) Distribution of and education on the daily use of insecticide-treated bed nets

D) Interfamily strain

21.The nurse assesses a 77-year-old woman who has been caring for her husband who has Alzheimer disease in their home for the past 8 years. The client complains of exhaustion and depression. Which stressors should the nurse most suspect as the cause of this client's symptoms? A) Marital strain B) Caregiver burden C) Spillover D) Interfamily strain

D) 10

21.The nurse is conducting research on the relative risk of acquiring lung cancer as a result of being exposed to secondhand smoke. The nurse is studying two different groups. Group A includes 400 adults who do not smoke but are exposed to secondhand smoke in their home on a daily basis. Group B includes 400 adults who do not smoke and are not exposed to secondhand smoke in their home. Over the course of 10 years, 20 subjects in Group A are diagnosed with lung cancer, whereas only 2 subjects in Group B are diagnosed with lung cancer. Which is the relative risk ratio that would result from this study? A) 0.5 B) 1 C) 5 D) 10

C)Arrive late for the appointment, after completing whatever tasks the client feels need to be accomplished first

21.The nurse is scheduling a follow-up appointment for a client who recently immigrated from Nicaragua. The client agrees to a 9:00-AM appointment on the following Wednesday. The nurse should anticipate that on the following Wednesday, this client will: A)Show up half an hour before the scheduled time B)Not return at all for the follow-up appointment C)Arrive late for the appointment, after completing whatever tasks the client feels need to be accomplished first D)Arrive exactly on time

A) Fluid -resistant gown B) Gloves D) Mask E) Goggles

21.The nurse prepares to enter the room of a client with suspected Ebola virus disease (EVD). Which pieces of personal protective equipment (PPE) should the nurse wear? (Select all that apply.) A) Fluid -resistant gown B) Gloves C) Respirator D) Mask E) Goggles

C)Develop evaluation questions

21.Which is the first step in developing a plan for evaluating a community-level intervention? A)Determine indicators or measures to answer evaluation questions B)Decide what method you will use to collect data C)Develop evaluation questions D)Decide how you will communicate your results

A,D,E

3. The case manager works with a client who has type 2 diabetes and heart disease. In this role, which actions should the nurse take in managing the client's diseases? (Select all that apply.) A) Provide weekly coaching sessions by telephone for blood glucose management. B) Supervise cardiac stress tests in the exercise room of the local hospital. C) Administer emergency insulin when the client arrives at the hospital in a diabetic coma. D) Demonstrate use of a glucose meter to the client. E) Review heart-healthy and diabetic-friendly food options with the client and his family.

A, B, D

3. The nurse is interested in taking on a role in the state's political decision-making process. Which political skills will the nurse need to be effective in this role? (Select all that apply.) A)Ability to understand others' behaviors B)Social skills C)Diagnostic skills D)Ability to network E)Ability to lie convincingly

B)Learned behavior

3. The nurse is present when a mother begins cupping her child. She states that it will help to heal the child's respiratory tract infection. The nurse understands that the cultural belief that cupping will aid in the healing process is a: A)Shared instinct B)Learned behavior C)Private idiosyncrasy D)Genetically programmed idea

B) To identify family health practices within a social and historical context

3. There are several approaches to family assessment that use particular concepts that nurses typically encounter in practice. These models provide an organized framework with which to conduct a family assessment. Which is the aim of the family assessment in the Life Span Development approach? A) To identify family routines and rituals that could influence family health B) To identify family health practices within a social and historical context C) To assess health patterns in 11 areas of family health D) To engage the family as a way to identify its strengths, with attention to family dynamics and concerns related to health and illness

C)All of the company's employees and include those living outside the community as part of the aggregate

3. While completing a community assessment, the nurse notes that the computer companies in the community hire a disproportionate number of their employees from local suburban communities rather than the community in which the company is housed. To create an accurate assessment, the nurse should collect data on: A)All of the company's employees but count those living outside the community separately, as outliers B)Only the employees that live in the community, disregarding the others as statistically irrelevant C)All of the company's employees and include those living outside the community as part of the aggregate D)Only the employees that live in the community in your initial assessment; then collect data on the others as part of a separate assessment for the community in which they live.

A)Develop an asthma education and surveillance program that addresses students, families, and faculty members.

3.Absenteeism due to illness has increased recently in the local middle school. The increase is noted to be related to exacerbation of asthma symptoms in the student population. Which would be the most effective approach in decreasing absenteeism among this population? A)Develop an asthma education and surveillance program that addresses students, families, and faculty members. B)Develop a policy to exclude students with a diagnosis of asthma from admission to the school. C)Create an asthma prevention plan within the school environment, based in the school nurse's office. D)Create an education program for faculty related to the prevention of exacerbation of asthma systems.

B) Every day 800 women die due to complications of pregnancy and childbirth.

3.The nurse prepares a presentation about the World Health Organization's World Health Statistics Report for 2013. Which trend is mentioned in this report? A) Children in low-income countries are 2 times more likely to die before reaching the age of 5 years than children in high-income countries. B) Every day 800 women die due to complications of pregnancy and childbirth. C) Zimbabwe now has a higher life expectancy than 7 out of 10 countries in Eastern Europe. D) Almost 5% of the world's adult population has diabetes.

B) The unexposed population is not likely to contract the illness

3.Three years after exposure to a virulent form of the flu, a population faces the same flu strain. The public health department recognizes that there are members of the population who have not been exposed to the flu strain. Which is most likely to happen? A) The unexposed population will contract Page 64 the illness B) The unexposed population is not likely to contract the illness C) The unexposed population will contract the illness and reinfect others D) The flu will reach epidemic proportions and both populations will become ill

B) Agency that acquires resources to help others from private rather than public sources

3.Which best describes a nongovernmental organization? A) Agency that is based in only one country but provides services to many other countries B) Agency that acquires resources to help others from private rather than public sources C) A group that collects data related to healthcare use across a variety of professional and service parameters D) An organization that uses endowed funds or private fundraising to address the needs of individuals, families, and populations

A) Number of deaths of children <1 year of age in one year / Number of live births in the same year X 1000

3.Which formula should be used to calculate the infant mortality rate? A) Number of deaths of children <1 year of age in one year / Number of live births in the same year X 1000 B) Number of fetal deaths plus infant deaths <7 days of age in one year/ Number of live births plus fetal deaths in the same year X1000 C) Number of deaths of children <28 days of age in one year/Number of live births in the same year X1000 D) Number of fetal deaths in one year/ Number of live births plus fetal deaths in the same year X 1000

A) Hippocrates of Cos

3.Who is considered the first epidemiologist? A) Hippocrates of Cos B) Aretaeus the Cappadocian C) Claudius Galen D) Susruta

A)Cross-cultural nursing

4. A Mexican-American client arrives at the emergency department. He reports abdominal pain and has been vomiting for 5 days. The nurse notes that he is exhibiting symptoms of dehydration, and his vital signs and labs confirm this. The nurse asks the client why he waited so long to seek medical care. He states that he is being treated by the Curandero and wishes to continue while in the hospital. The nurse recognizes the different cultural assumptions regarding care. This situation is an example of: A)Cross-cultural nursing B)Ethnocentrism C)Cultural safety D)Transcultural bias

C)Interview members of the neighborhood to determine why they do not use the clinic.

4. Multiple family units in an underserved neighborhood have children who have not been vaccinated per health guidelines. The nurse researches the problem and establishes a well-child clinic within walking distance to the neighborhood. The clinic is poorly attended. Which would be the nurse's best action to revise the program and improve attendance to the clinic? A)Determine whether the location of the clinic is not conducive to attendance. B)Discuss the problem of attendance at the next community council meeting. C)Interview members of the neighborhood to determine why they do not use the clinic. D)Discuss the problem with the local minister and request him to discuss the clinic at the next prayer service.

B) Advocacy

4. The case manager works with a 10-year-old girl who has asthma. Because the client's family has trouble affording inhalers, the case manager researches client assistance programs through which they could receive free medications. This aspect of case management is known as: A) Care management B) Advocacy C) Disease management D) Interdisciplinary collaboration

A,B,D

4. Which changes in the environment (second level from the bottom of the health impact pyramid) make the choice for a healthy behavior the default or easy choice? (Select all that apply.) A)Iodization of salt B)Restrictions on smoking in public places C)Community immunization programs D)Changing food manufacturing to eliminate trans fats E)Treatment of hypertension

D) 1) Setting an agenda, 2) policy formulation, 3) policy adoption, 4) policy implementation, 5) policy assessment, 6) policy modification

4. Which represents the correct order of events in the policy-making process? A) 1) Policy formulation, 2) policy assessment, 3) policy adoption, 4) policy implementation, 5) policy modification, 6) setting an agenda B) 1) Setting an agenda, 2) policy formulation, 3) policy assessment, 4) policy modification, 5) policy adoption, 6) policy implementation C) 1) Policy formulation, 2) setting an agenda, 3) policy implementation, 4) policy modification, 5) policy assessment, 6) policy adoption D) 1) Setting an agenda, 2) policy formulation, 3) policy adoption, 4) policy implementation, 5) policy assessment, 6) policy modification

A) Age B) Sex D) Physical and emotional health E) Immune status

4.In infectious disease epidemiology, just the presence of an infectious agent is not sufficient to produce an infectious disease. Which host factors determine whether a person is at risk for an infection or an infectious disease? (Select all that apply.) A) Age B) Sex C) Occupation D) Physical and emotional health E) Immune status

C) Share any genograms and ecomaps that illustrate relationships with family members

4.The community health nurse has limited time for family assessments because of demanding caseloads or staffing shortages. Using your knowledge that focused family interviews of 15 minutes or less can yield a wealth of information, which is an example of an activity that will acknowledge the family's strengths? A) Ask family members if they see an area that could be changed B) Allow everyone present to voice observations, insights, or concerns before offering how they could change the situation C) Share any genograms and ecomaps that illustrate relationships with family members D) Collaborate with the family to set priorities, plan care, and evaluate goals

A) Statistical procedure that removes the effects of differences in the composition of a population, such as age, when comparing one with another

4.Which correctly defines an adjusted rate? A) Statistical procedure that removes the effects of differences in the composition of a population, such as age, when comparing one with another B) Measure of the probability that people without a certain condition will develop that condition over a period of time C) Measure of the number of people in a given population who have an existing condition at a given point in time D) Detailed rates that are calculated using the number of people in the smaller subgroups of the population in the denominator

A) Temperature C) Precipitation D) Humidity

4.Which environmental factors are likely to affect the life cycle of disease pathogens and their vectors and thus disease outbreaks? (Select all that apply.) A) Temperature B) Geology C) Precipitation D) Humidity E) Amount of daylight

D) A tentative hypothesis is formulated

4.Which occurs in the epidemiologic process that is similar to the diagnosis phase of the nursing process? A) Data are gathered from reliable sources B) Healthcare needs and assets are identified C) Goals and objectives for care are established D) A tentative hypothesis is formulated

A) Burning of wood and dung for cooking and heat B) Low status of women D) Not wrapping an infant immediately after birth

4.Which risk factor for poor health is associated with low- and middle-income countries? (Select all that apply.) A) Burning of wood and dung for cooking and heat B) Low status of women C) Sedentary lifestyle D) Not wrapping an infant immediately after birth E) Overconsumption of fatty foods

A,C,D

5. At an international nurses' conference, nurses from all around the world make contact. The various communities represented at this conference most likely: (Select all that apply.) A)Vary significantly in composition B)Have no shared interests and goals related to health and safety C)Are challenged in their attempts to identify its members D)Thought of as members of a global community E)Do not need to assess their communities' needs because they are obvious

A) Primary

5. The local clinic is dedicated to the well adult and child. It has evening hours and offers varied programs for the community. The programs include immunizations and classes on fire safety, health education, and car safety, to name a few. How would the nurse explain the level of prevention used in this setting to your colleagues? A)Primary B)Secondary C)Tertiary D)Quaternary

C)Competence

5. The nurse conducts a well-child clinic in your community. The client population is culturally diverse, and, although the clinic is busy, it runs smoothly because it has an interpreter. In addition to the interpreter, it is important that the nurses treat each client that reflects his or her individual cultural needs. This is cultural: A)Bias B)Artwork C)Competence D)Beliefs

C)Setting an agenda

5. The nurse meets with a group of fellow nurses. They discuss the number of children who have died in hot cars and how likely they are to gain support from their state legislators to address the problem. The nurses are currently in which step of the policy-making process? A)Policy formulation B)Policy adoption C)Setting an agenda D)Policy assessment

A,E

5. Which exemplify clinical interventions (the level just above the middle of the health impact pyramid)? (Select all that apply.) A)Administration of insulin injections for diabetes B)Restrictions on smoking in public places C)Community immunization programs D)Instruction provided by a nurse regarding proper nutrition E)Treatment of hypertension

A) Poor hygiene C) Poor sanitation

5.Tent communities are constructed in a rural community by the American Red Cross after a disaster. Because of overcrowding, surrounding states create accommodations for the displaced and homeless. The survivors are accommodated in the cities of four Southern states. Which factors may increase the possibility of disease emergence in the cities? (Select all that apply.) A) Poor hygiene B) Decreased socialization C) Poor sanitation D) Decreased viral load E) Increased violence

D) Interdisciplinary collaboration

5.The case manager for a 12-year-old boy who is obese and prediabetic. To help the client develop a diet that will promote weight loss and control of his glucose level, the case manager decides to consult with a dietician, who provides several current journal articles that present findings on various diets. This aspect of case management is known as: A) Care management B) Advocacy C) Disease management D) Interdisciplinary collaboration

A,B,C

5.The community health nurse is reviewing guidelines for the 15-minute assessment to keep the interview focused and family centered. Which questions and considerations for the family assessment can be asked for the question "Who is part of the family?" (Select all that apply.) A) What is the nature of individuals' connections to one another? B) Does the individual live alone or have no living family members? C) What influences from the family of origin are present in daily life? D) Do members speak independently or is an interpreter present? E) What observations can be made about the interpersonal dynamics between members?

B) Distribute a mosquito bed net to every home in the nation. D) Increase the number of girls who attend primary schools by 100%. E) Provide training on prevention of postpartum hemorrhage to all healthcare workers.

5.The nurse is assigned to prepare the yearly goals for the community health agency in a developing nation. Which goals, on the basis of the United Nations' Millennium Development Goals for improving global health, should the nurse include? (Select all that apply.) A) Initiate an afterschool sports program at every school in the nation to increase physical activity among children. B) Distribute a mosquito bed net to every home in the nation. C) Provide 1 million free allergy shots to people at highest risk. D) Increase the number of girls who attend primary schools by 100%. E) Provide training on prevention of postpartum hemorrhage to all healthcare workers.

C) Incubation period

5.When an infectious agent enters the host and begins to multiply, an infection occurs. The time between exposure to an infectious agent and the manifestation of symptoms in the host is known as: A) Carrier time B) Colonization C) Incubation period D) Latent period

C) Prevalence

5.Which rate measures the number of people in a given population who have influenza at a given point in time? A) Adjusted B) Incidence C) Prevalence D) Specific

A) William Farr

5.Who in the history of epidemiology developed mortality surveillance systems, addressed basic epidemiologic concepts, and is considered the founder of modern statistics? A) William Farr B) John Graunt C) Florence Nightingale D) John Snow

A) United States Department of Health and Human Services

6. A nurse is coordinating an initiative in the community to make sure that the healthcare needs of local refugees and asylees are recognized and addressed. Which federal agency should this nurse contact, as it is directly involved with the health and healthcare of refugees and asylees in the United States? A) United States Department of Health and Human Services B) National Institutes of Health C) Agency for Healthcare Research and Quality D) Centers for Disease Control and Prevention

C,D,E

6. A student is considering becoming a home health nurse. In researching this type of nursing, the student finds that several key factors have played a part in the development of home healthcare as it is today? These include: (Select all that apply.) A) Increase in the number of hospitals nationwide B) Reduction in the incidence of chronic diseases C) Development of health insurance D) Rising costs in healthcare E) Increase in medical and nursing specialization

B)Policy adoption

6. The hospital board is in the process of developing a policy regarding the use of cell phones within the hospital. At the current meeting, the board has decided to allow use of cell phones only within certain waiting areas within the hospital, and nowhere else. Which stage of policy-making does this action represent? A)Policy formulation B)Policy adoption C)Policy implementation D)Policy assessment

C)Blood pressure for children aged 12 to 18 years in Marks School will be reduced by 5% after petting the dog for 1 day in March 2019.

6. The nurse prepares community program objectives that are specific, measurable, achievable, relevant, and time bound (SMART) to help in planning interventions and establishing measurement systems to evaluate programs and outcomes. Which is a measurable objective? A)Program will reduce teenage obesity. B)Fewer teens will start smoking. C)Blood pressure for children aged 12 to 18 years in Marks School will be reduced by 5% after petting the dog for 1 day in March 2019. D)Number of older minority residents in Legacy Independent facility receiving a flu shot will triple.

An older client is seeking a fourth refill on a prescription for benzodiazepines. The nurse is concerned that the client may be developing a tolerance for the drug. Which comment by the client would most confirm this concern? A)"Lately I've had to double up on my dose just for it to be effective." B)"I ran out of pills a week ago and have felt sweaty and agitated ever since." C)"I feel like I just can't function without the medicine." D)"The medicine has been wonderful. I think one more refill should do it for me."

A) "Lately I've had to double up on my dose just for it to be effective."

9.The community health nurse addresses cyber bullying in the local middle schools. Currently, there are no policies in place for this issue. The most appropriate first step is to: A) Explore current programs to prevent bullying and youth violence B) Develop rules for acceptable and safe use of all electronic media for all students C) Ban use of the Internet at all schools D) Actively monitor students' social media postings

A) Explore current programs to prevent bullying and youth violence

1. A nurse is striving to practice patient-centered care at a hospital. Which action best exemplifies providing patient-centered care? A) Having a client complete a self-reported functional status indicator and then reviewing it with the client B) Explaining to a client the benefits of computer-assisted robotic surgical techniques, which the hospital recently implemented C) Recording a client's signs and symptoms in an electronic health record D) Performing continuous glucose monitoring of a client while the client is in the hospital

A) Having a client complete a self-reported functional status indicator and then reviewing it with the client

10.Which are advantages of disaster management JITT for nurses? (Select all that apply.) A)Is cost-effective because it requires the least amount of resources to produce the final result B)Provides the nurse with comprehensive knowledge of how to respond to many different types of disaster C)Shortens the period of time between learning and application D)Includes classroom sessions and on-site presentations E)Is offered online for easy access

A)Is cost-effective because it requires the least amount of resources to produce the final result C)Shortens the period of time between learning and application D)Includes classroom sessions and on-site presentations E)Is offered online for easy access

The nurse performs motivational interviewing with a college student in the university health center regarding the student's history of binge drinking. The nurse hopes to ascertain the student's readiness to change her behavior and encourage her to make better choices related to alcohol use. The interview should begin with: A)Listening empathetically to establish trust B)Eliciting feelings about how the drinking behavior is affecting the student's life C)Noting discrepancies between attitudes toward alcohol and alcohol-associated harms D)Being supportive of the student's life goals

A)Listening empathetically to establish trust

7. The nurse is a member of the emergency and disaster preparedness committee for an organization. The nurse's role is to head the data collection subcommittee. What are the key areas that would be included in your plan? A)Morbidity, mortality, mental health, and infectious disease B)Morbidity, mortality, displacement, and community infrastructure C)Morbidity, mortality, mental health, and chronic illness D)Morbidity, mortality, displacement, and healthcare workers

A)Morbidity, mortality, mental health, and infectious disease

13.The nurse is in charge of implementing a personal protective equipment (PPE) program at the hospital, in response to a chemical terrorist attack. Which components should be included in this program? (Select all that apply.) A)Selection, maintenance, and use of PPE B)Training of employees in PPE use C)Identification of hazards present D)Identification of commodities to be distributed via POD emergency supply centers E)Clinical assessment and triage drills

A)Selection, maintenance, and use of PPE B)Training of employees in PPE use C)Identification of hazards present

2. A nurse is caring for an older client who is struggling to manage her type 2 diabetes mellitus. The nurse should recognize which social determinants of this client's health? (Select all that apply.) A) Household income of $23,000 per year B) Reading level of a third grader C) Medication ineffective due to error in prescription D) Originally from Sudan E) No family in the area

A, B, D, E

2.In an institution-based model of nursing, the nurse: (Select all that apply.) A) Serves a health system with assignment to particular congregational settings B) Serves as a liaison between a health system and a congregation C) Is part of the ministry staff of the congregation D) Serves a particular faith community by virtue of a contract or job description E) Provides care of the human spirit

A,B A) Serves a health system with assignment to particular congregational settings B) Serves as a liaison between a health system and a congregation

15.Personal responsibility for health involves active participation in one's own health through education and lifestyle changes. Which exemplify personal responsibility? (Select all that apply.) A)Reviewing one's own medical records B)Monitoring the positive and negative effects of prescription and over-the-counter medications C)Avoiding tobacco and recreational drug use D)Showing up for scheduled tests and procedures E)Eating the types of foods one most enjoys

A,B ,C,D

21. The nurse cares for a client whose husband of 50 years died a year ago. A behavior that indicates the client has reached the reorganization phase of grief is when she: A) Joins a book club to get out of the house and meet new people B) Expresses no emotions regarding the loss of her husband C) Begins crying on a daily basis D) Experiences panic attacks regularly

A) Joins a book club to get out of the house and meet new people

The nurse visits the home of client in a low-income community and notices that paint is flaking off the walls and forming a dust in the corners of some rooms. The nurse asks the client when the house was built, and she responds that it was built in 1959. Which contaminant should the nurse expect to find in this home? A) Lead B) Radon C) Asbestos D) Pesticide

A) Lead

7. A line operator in a manufacturing facility comes in to the on-site clinic reporting numbness in his hands after excessive exposure to the toxin benzene. Applying the epidemiologic triad to this scenario, which is the host? A) Line operator B) Manufacturing facility C) Benzene D) Hands

A) Line operator

19. A woman mentions that her 6-month-old son has demonstrated some odd behavior. She thinks it might be related to autism. She asks what age would be best to have screening done on the child. The nurse should respond: A) Now, as an infant B) When the child begins to walk C) When the child is preschool-aged D) When the child starts school

A) Now, as an infant

6.The nurse cares for a client whose husband of 50 years died last week. The nurse arrives at her home to find her staring out of a window in a stupor, her face devoid of any emotion. When the nurse speaks with her, the client seems withdrawn and distant. Which phase of grief should the nurse most suspect this client to be in? A) Numb shock B) Emotional turmoil C) Reorganization D) Resolution

A) Numb shock

The nurse asks a client, "Do you wear the clothes you wear at work to home?" This is an example of which subject on the exposure history? A) Present work B) Past work C) Home/residence D) Concerns

A) Present work

9.The nurse works with a variety of clients at a general hospital in a rural area. For which clients would it be most appropriate to administer palliative care? (Select all that apply.) A) 92-year-old man who has just suffered a stroke B) 83-year-old woman with end-stage dementia C) 12-year-old girl with terminal lymphoma D) 74-year-old man with osteoarthritis E) 54-year-old woman with high blood pressure

A,B,C A) 92-year-old man who has just suffered a stroke B) 83-year-old woman with end-stage dementia C) 12-year-old girl with terminal lymphoma

8.An advisory board is essential for solving problems and avoiding pitfalls in faith communities. How might the advisory board offer help to a faith community nurse? (Select all that apply.) A) Offer leadership B) Establish policy C) Develop guidelines D) Establish the budget for the program E) Provide recommendations on clinical best practices

A,B,C A) Coordinate the discharge and transition of congregation members from the hospital to home C) Recommend services that the hospital provides D) Serve as a health advisor to members of the congregation

2.On the basis of the eight recommended components of a comprehensive school health program, which should be included in a K-12 health education curriculum? (Select all that apply.) A) Consumer health education B) Environmental health education C) Sexuality education D) Nutrition counseling E) Physical development activities

A,B,C,D A) Consumer health education B) Environmental health education C) Sexuality education D) Nutrition counseling

3.Which services are reimbursable by Medicaid? (Select all that apply.) A) Speech/language therapy and evaluations B) Occupational therapy and evaluations C) Physical therapy and evaluations D) Nutrition screening and evaluations E) Allergy testing and evaluations

A,B,C,E A) Speech/language therapy and evaluations B) Occupational therapy and evaluations C) Physical therapy and evaluations E) Allergy testing and evaluations

5.Which exemplify biological hazards? (Select all that apply.) A) Contaminated body fluids B) Poisonous plants C) Diesel exhaust D) Venomous snakes E) Aerosols

A,B,D A) Contaminated body fluids C) Diesel exhaust D) Venomous snakes

7. The school nurse at a public high school recently noticed an increase in students smoking before and after school. Using the community school model to reduce tobacco use among your students, which interventions are appropriate? (Select all that apply.) A) Starting an after-school tobacco cessation program with the assistance of local representatives of the American Cancer Society B) Having a local clinical oncologist speak at a PTA meeting about the hazards of tobacco use among children C) Confronting groups of students while they are smoking with information regarding the school's smoking policy and the dangers of smoking D) Promoting the community's "great smoke-out" smoking cessation event each year E) Encouraging the school's administration to designate a single smoking area outside to limit nonsmokers' exposure to the smoke

A,B,D A) Starting an after-school tobacco cessation program with the assistance of local representatives of the American Cancer Society B) Having a local clinical oncologist speak at a PTA meeting about the hazards of tobacco use among children D) Promoting the community's "great smoke-out" smoking cessation event each year

8.An elementary school student comes to school nurse's office reporting a headache. The nursing assessment begins by gathering which subjective data? (Select all that apply.) A) Where it hurts on the child's head B) Whether the pain is constant, throbbing, or intermittent C) What the child's temperature is on arrival D) How bad the pain is on a scale of 1 to 10 E) The presence of a bruise on the child's forehead

A,B,D A) Where it hurts on the child's head B) Whether the pain is constant, throbbing, or intermittent D) How bad the pain is on a scale of 1 to 10

1.Hospice care includes services that are reasonable and necessary for the comfort and management of a terminal illness. Which services might be included? (Select all that apply.) A) Physician services B) Nursing care C) Long-term inclient pain control D) Hospice aide services E) Social worker services

A,B,D,E A) Physician services B) Nursing care D) Hospice aide services E) Social worker services

5.The nurse provides palliative care to a client with end-stage renal disease and a chronic upper respiratory infection. Which interventions should be included in this type of care? (Select all that apply.) A) Providing the client with a foot massage B) Administering ordered analgesics C) Assisting with a kidney transplantation D) Clearing secretions from the client's airway for easier breathing E) Gently stretching the client's arms and legs to preserve range of motion

A,B,D,E A) Providing the client with a foot massage B) Administering ordered analgesics D) Clearing secretions from the client's airway for easier breathing E) Gently stretching the client's arms and legs to preserve range of motion

19.Which strategies have been shown successful for preventing adolescent pregnancy? (Select all that apply.) A) Responsible sexual behavior education B) Abstinence education C) Mandated religious education on sex practices D) Contraceptive counseling E) Confidential reproductive services

A,B,D,E A) Responsible sexual behavior education B) Abstinence education D) Contraceptive counseling E) Confidential reproductive services

11.Pain and symptom control are crucial to the delivery of quality end-of-life care. For which signs of pain should the nurse carefully assess? (Select all that apply.) A) Grimacing or strained facial expression B) Slow heartbeat C) Diaphoresis D) Moaning or groaning at rest or movement E) Taking sips of water from a cup

A,C,D A) Grimacing or strained facial expression C) Diaphoresis D) Moaning or groaning at rest or movement

15.The nurse assesses a 12-year-old boy who has been diagnosed with bipolar disorder. Which behaviors should the nurse most expect to observe in this child? (Select all that apply.) A) Irritability B) Distractibility C) Screaming D) Rage episodes E) Sleep disturbances

A,C,D,E

20. The occupational health nurse is a member of the emergency planning committee in the workplace. The nurse is helping to formulate an emergency plan for the facility. Which key components should the nurse include in the emergency plan? (Select all that apply.) A) Alarms B) Phones C) Reporting D) Communication E) Evacuation

A,C,D,E A) Alarms C) Reporting D) Communication E) Evacuation

19.The occupational health nurse is a member of the emergency planning committee in the workplace. The nurse is helping to formulate an emergency plan for the facility. Which goals should the nurse keep in mind while developing the plan? (Select all that apply.) A) Establish clear reporting instructions for employees. B) Provide instructions on safe lifting techniques. C) Identify hazardous substances that workers are exposed to in their daily work. D) Name key personnel who will assume necessary tasks. E) Establish emergency escape routes.

A,D,E A) Establish clear reporting instructions for employees. D) Name key personnel who will assume necessary tasks. E) Establish emergency escape routes.

13.A newly hired health promotion specialist works at a meat-processing facility. The specialist is in charge of developing a program that encourages a balance among work, family, personal, health, and psychosocial concerns. Which components are appropriate to include in this program? (Select all that apply.) A) Signage that demonstrates safe lifting techniques B) Free screening for HIV C) Diagnosis of potentially malignant skin lesions D) Clinic on proper use of safety goggles and other equipment E) Monitoring of noise levels within the facility

A,D,E A) Signage that demonstrates safe lifting techniques D) Clinic on proper use of safety goggles and other equipment E) Monitoring of noise levels within the facility

The information gained from environmental epidemiologic studies can be very helpful in trying to identify whether an exposure or what particular exposures have made people ill. Major challenges to most environmental epidemiology studies include: (Select all that apply). A) Resource intensive in terms of personnel and money B) Period between exposure and illness can be very short C) Weak ability to identify outbreaks of infectious disease D) Limited availability of data on many contaminants and their effect on people's health E) Time consuming to perform

A) Resource intensive in terms of personnel and money D) Limited availability of data on many contaminants and their effect on people's health E) Time consuming to perform

8.Which injury has the highest incidence rate among occupational injuries and illnesses that result in days away from work? A) Sprains B) Broken bones C) Cuts D) Carpal tunnel syndrome

A) Sprains

6.The nurse examines a client who is taking haloperidol for schizophrenia and notices that she habitually twists her tongue and arms. This side effect of her medication is known as: A) Tardive dyskinesia B) Akathisia C) Dystonia D) Neuroleptic malignant syndrome

A) Tardive dyskinesia

3. The management team is in the process of identification of hazards associated with the ice storm that is expected to disable electrical service to the city. To identify areas most vulnerable to damage and plan for an effective response, which data or methods of data collection may be used? (Select all that apply.) A)Geographic information B)Satellite imagery C)Census data D)Historical data on previous events E)Planning board declarations

A)Geographic information B)Satellite imagery D)Historical data on previous events

In assessing a 13-year-old Native American boy who lives on a reservation, the nurse finds that he regularly drinks alcohol and smokes cannabis. He says that most of his friends do so, as well. The nurse is aware that this rate of prevalence is far higher than that for 13-year-olds of all ethnicities nationwide. Which social conditions have researchers associated with this health disparity? (Select all that apply.) A)Greater availability of cannabis from local farms B)Higher socioeconomic status C)Lower academic attainment D)Proximity to Mexico and drug gangs E)College enrollment

A)Greater availability of cannabis from local farms C)Lower academic attainment D)Proximity to Mexico and drug gangs

The nurse assesses a 33-year-old who abuses alcohol. To gauge how the client's substance use is impairing his ability to fulfill normal role functions, the nurse should use: (Select all that apply.) A)History of arrests for drunken driving B)Age at the time of first alcoholic drink C)Jobs lost due to intoxication at work and chronic absenteeism D)Failing grades in night classes at the local community college E)Blood alcohol level at time of last arrest for driving while intoxicated

A)History of arrests for drunken driving C)Jobs lost due to intoxication at work and chronic absenteeism D)Failing grades in night classes at the local community college

B)The hospital can more easily implement programs of care for the clients.

6. Using the tenets of secondary prevention, the committee seeks to establish an HIV/AIDS clinic. The nurse states that it is important for the location of the clinic to be associated with the local hospital. The committee states that the cost of the clinic associated with the hospital is higher than one that is free standing. Which presents the best rationale for the nurse's suggestion? A)Third-party insurance will reimburse the clinic visits only if the clinic is attached to the hospital. B)The hospital can more easily implement programs of care for the clients. C)The hospital will cover the clients only if the clinic is approved by the hospital governance board. D)The hospital will have a vested interest in the success of the clinic.

A) Airborne

6.A client with tuberculosis sneezes in the waiting room and infects several other clients who are sitting on the other side of the room. Which mechanism of transmission is involved in this case? A) Airborne B) Droplet C) Direct contact D) Indirect contact

A,C,D

6.A nurse, new to the community health agency, works in a culturally diverse area of the community. The nurse is responsible for providing holistic care to clients and to be culturally competent. The health agency requires the nurse to demonstrate which competency to exhibit cultural competence? (Select all that apply.) A)Value diversity B)Adopt the client's cultural values C)Acquire cultural knowledge D)Adapt to diversity E)Speak the language of the client

C) Adaptation

6.An 80-year-old client is being treated for chronic urinary tract infections. She has received multiple antibiotics in the past 6 months. The physician orders a urine culture. The results of the urine culture are as follows: Staphylococcus aureus >100,000 colonies and demonstrated resistance to sulfamethoxazole and trimethoprim (Bactrim), penicillin, methicillin, and erythromycin. The client is diagnosed with methicillin -resistant Staphylococcus aureus (MRSA) on the basis of the findings of her urine test. Based on the client's history, a cause of the MRSA might be microbial: A) Rejection B) Assimilation C) Adaptation D) Stagnation

C) A 63-year-old woman's sense of responsibility as the oldest among her siblings for taking care of her chronically ill parents

6.The community health nurse is assessing stressful life events in a family. Which exemplifies a life event related to illness and family care strains? A) A history of police reports of a 23-year-old man's hitting his girlfriend B) A couple in their 40s divorcing C) A 63-year-old woman's sense of responsibility as the oldest among her siblings for taking care of her chronically ill parents D) A 35-year-old woman not speaking to her sister for 5 years after not being invited to her wedding

C) Crude

6.The nurse is measuring the occurrence of chronic obstructive pulmonary disease being investigated in the entire population. Which rate does this represent? A) Adjusted B) Attack C) Crude D) Incidence

A)Policy modification

7. The hospital board is in the process of developing a policy regarding the use of cell phones within the hospital. After 6 months of a policy being in place and weeks of ongoing evaluation of the policy by individual board members, the board decided to expand the areas where cell phone use is permitted to include all waiting rooms and some client rooms. Which stage of policy-making does this action represent? A)Policy modification B)Policy adoption C)Policy implementation D)Policy assessment

A,B,D

7. The nurse works in a facility that provides hospice care for clients with cancer diagnoses. The nurse recognizes that the care provided in this facility is tertiary in nature. The nurse develops a plan for the clients that would include the tenets of tertiary prevention. The plan would include: (Select all that apply.) A)Palliative care B)Pain control C)Isolation D)Supportive spiritual development E)Diminished involvement of significant others

A,B,D

7. Which aspects of Lewin's change theory will help people visualize and create needed change? (Select all that apply.) A)Unfreezing the status quo B)Changing or moving to a new state C)Enforcing the status quo D)Refreezing to sustain the change or changes made E)Increasing restraining forces

D) To establish a free exercise program at a community center to promote fitness in a low-income neighborhood with high obesity rates

7. Which best exemplifies a public health goal in the United States? A) To restore the ability of an 80-year-old to perform activities of daily living after a stroke B) To lobby for legislation that ensures the accessibility to health insurance on the basis of one's health C) To develop treatments to address symptoms related to chemotherapy D) To establish a free exercise program at a community center to promote fitness in a low-income neighborhood with high obesity rates

C) Study of the distribution and determinants of states of health and illness in human populations

7.The community health nurse knows that early attempts to understand illness and disease focused on the study of the experiences of individual people. Using this knowledge, how would the nurse define epidemiology to a group of nursing students? A) An outbreak that occurs when there is an increased incidence of a disease beyond that which is normally found in the population B) Model based on the belief that health status is determined by the interaction of the characteristics of the host, agent, and environment C) Study of the distribution and determinants of states of health and illness in human populations D) Epidemiologic model that strongly emphasizes the concept of multiple causation while de-emphasizing the role of agents in explaining illness

B) Enhanced infection control measures

7.The community health nurse works in a region where severe acute respiratory syndrome (SARS) has just been reintroduced into the environment. Which measure is imperative for the nurse to take to prevent spread of the disease? A) Prophylactic antibiotics B) Enhanced infection control measures C) Use of alcohol cleansers D) Report of clusters to the Centers for Disease Control and Prevention

A,C,D

7.The nurse is called to make an unscheduled visit to a new client in her home. The purpose of the visit is to admit the client and treat an abdominal wound. The nurse discovers that the client is from a culture unfamiliar to her. In this situation, the nurse should demonstrate cultural competence and awareness by: (Select all that apply.) A)Being open to the client's ideas and way of life B)Obtaining objective data when caring for the client C)Exhibiting respect and patience D)Being aware of your own culture E)Assuming that you have enough knowledge to get by

D) Specific.

7.The nurse wishes to compare the number of persons aged 40 to 49 years who die each year with the midyear population of persons in this age range, per 100,000 population. Which type of rate is the nurse attempting to calculate? A) Adjusted B) Incidence C) Prevalence D) Specific

D) Water

7.The student-nurse discusses transmission that involves contact of a susceptible host with a vehicle. Based on the chain of infection links, which exemplifies a vehicle? A) Animals B) Insects C) Reptiles D) Water

A) First-degree relatives who live together

7.Which exemplifies a nuclear family? A) First-degree relatives who live together B) First- and second-degree relatives who live together C) First- and second-degree relatives who live in the same neighborhood D) Individuals who are not blood relatives but share a common locale of origin or culture

A) Tobacco use B) Insufficient physical activity D) High blood pressure

7.Which key factors contribute to the rise of noncommunicable diseases in low- and middle-income countries? (Select all that apply.) A) Tobacco use B) Insufficient physical activity C) Unclean water D) High blood pressure E) Lack of contraceptive use

B)Developing culturally relevant and gender-sensitive interventions

8. An immigrant from Guatemala, age 47, is admitted to the coronary care unit with congestive heart failure. This is the fourth admission for this client, and the physician believes that the client's inability to adhere to his diet and medication regimen is the reason for his frequent admissions. The physician refers the client to the local community health nurse, on discharge from the hospital. Which would be most appropriate for the nurse to include in this client's plan of care? A)Insisting that the client adhere to his diet B)Developing culturally relevant and gender-sensitive interventions C)Teaching the family to adapt their cultural dietary plans to the client's diagnosis D)Assessing the client's financial needs and obtaining financial assistance as needed

A) Individual care with a focus on cure

8. In the United States, the system of healthcare has historically given and continues to give stronger support to which types of care? A) Individual care with a focus on cure B) Community care with a focus on cure C) Individual care with a focus on prevention D) Community care with a focus on prevention

C)Develop school-based education programs for both the children and their caregivers

8. The community health nurse is responsible for the general health of the community on multiple levels. The nurse discovers an increase in the number of sick days used by children aged 7 to 11 years at the local elementary school in recent weeks. Research indicates that the families of asthmatic children tend to be less educated and less likely to comply with current treatment plans. Using the epidemiologic approach, the next step in the care plan should be to: A)Conduct a broad-based survey of all families with asthmatic children within several communities B)Determine trends that interfere with family problem-solving techniques C)Develop school-based education programs for both the children and their caregivers D)Submit a proposal to the town for neighborhood asthma clinics

B,C,E

8. The nurse applies a cost-benefit analysis to the healthcare model. Which costs are the primary concerns of the client? (Select all that apply.) A) Total payment for the service B) Out-of-pocket payment C) Suffering and pain D) Actual and direct cost of providing a service E) Psychological costs

C)Identifying the forces driving the change and those resisting it

8. Which most accurately describes the purpose of force field analysis? A)Unfreezing the status quo B)Changing or moving to a new state C)Identifying the forces driving the change and those resisting it D)Refreezing to sustain the change or changes made

B) Cause-specific mortality rate

8.. The nurse wishes to determine how many people in the city die from heart attacks each year per 100,000 population. Which type of rate is the nurse attempting to calculate? A) Age-specific mortality rate B) Cause-specific mortality rate C) Case fatality rate D) Crude mortality rate

A) Endemic

8.Chloroquine-resistant malaria occurs in most of Africa, the Middle East, and Asia and all of the South Pacific islands. This type of disease occurrence is known as: A) Endemic B) Epidemic C) Pandemic D) Outbreak

A,C,E

8.Reasons blended families are becoming more widespread include an increase in the number of: (Select all that apply.) A) People who remarry following a divorce B) People who remain single after being widowed C) Recognized gay and lesbian relationships D) Married couples having children E) Couples cohabiting with children from prior relationships

C)Consulting with the healer and updating the physician regarding treatment and any herbal medications that may be used

8.The nurse is caring for a client who is a refugee from Haiti. The client's family has been displaced because of an earthquake in their country. The client has healing fractures on his right arm as well as fractured ribs. He tells the nurse that, before care begins, the nurse must consult with his Voodoo healer. Which is the nurse's best response? A)Teaching the client that he is in the United States and should abide by Western medical practices B)Informing the physician that the client is noncompliant C)Consulting with the healer and updating the physician regarding treatment and any herbal medications that may be used D)Encouraging the client to use a complementary approach to his care

C) Proprietary

8.The nurse recently took a position in a national chain of home healthcare agencies directed at caring for clients who are on home hemodialysis. The services provided are often paid for privately by families, and any profit margin is used to benefit the owner of the agency. Which type of agency is this? A) Private/voluntary B) Hospital based C) Proprietary D) Official

D) 870 million people still do not consume enough food to meet their nutritional energy requirements.

8.The parish community health nurse is assigned to prepare the goals for a 6-month mission trip to a developing nation. Which challenge to the Millennium Development Goals should the nurse consider? A) Child mortality has increased 21% since 1990. B) The percentage of people living on less than $0.25 a day rose from 22% in 1990 to 47% in 2010. C) The number of children out of school worldwide increased from 57 million to 102 million. D) 870 million people still do not consume enough food to meet their nutritional energy requirements.

B) Respiratory hygiene C) Hand hygiene D) Droplet precautions for visitors in contact

8.There is an outbreak of Middle East respiratory syndrome coronavirus (MERS -CoV) in the community. The nurse is responsible for the dissemination of the education plan adopted by the public health department. Key components of the plan should include: (Select all that apply.) A) Quarantining infected families in their homes B) Respiratory hygiene C) Hand hygiene D) Droplet precautions for visitors in contact with affected clients E) Contact precautions for all cases

B) Primary measurement used to describe either the occurrence or the existence of a specific state of health or illness

8.Which most accurately defines ìrateî? A) Course of a disease or condition from the onset to resolution B) Primary measurement used to describe either the occurrence or the existence of a specific state of health or illness C) Probability or likelihood that a disease or illness will occur in a group of people who presently do not have the problem D) Characteristic or event that has been shown to increase the probability that a specific disease or illness will develop

D) $140

9. A client visits the office of an independent nurse practitioner to have routine blood work done. The client returns 1 week later for a follow-up visit to discuss the results and formulate a new treatment plan. On the basis of the client's insurance plan, the client is only obligated to pay $15 copay for this service. The total payment that the client's insurance company provides the practitioner is $180, which covers the $80 fee for a 15-minute visit and the $100 cost of laboratory services. On the basis of the nurse's current client load, $40 of the $80 fee for the visit covers overhead and the remaining $40 is profit. In terms of cost-benefit analysis, which is the actual financial cost to the nurse for providing this service? A)$100 B)$15 C)$180 D)$140

C) TRICARE

9. A nurse works predominantly with clients who are active military personnel. Which publically funded program provides health benefits to this population? A) Medicare B) Medicaid C) TRICARE D) CHIP

C)Encourage the group to participate in the decision making

9. The nurse conducts a class on the basics of nutrition to a group of obese adults. The nurse counsels the audience, telling them that they need to change their behaviors. By the end of the class, the nurse has lost the audience. To enhance audience participation and learning in the next class, the nurse should: A)Use consistent methods of persuasion B)Offer advice while the participants are passive C)Encourage the group to participate in the decision making D)Encourage the group to continue ambivalent behaviors

B)Research the vaccine compliance in the school

9. The school nurse notifies the community nurse that there have been five confirmed cases of pertussis in the sixth-grade classes. In addition to responding to the cluster of cases, the community nurse should immediately: A)Recommend that the school be closed B)Research the vaccine compliance in the school C)Recommend that notes be sent home to the parents regarding a pertussis epidemic D)Consult with the infection control physician at the local hospital

B) Hospital based

9. Which type of agency can be either profit or nonprofit? A) Private/voluntary B) Hospital based C) Proprietary D) Official

D) Antigenic shift

9.International health experts indicate that the avian flu is the virus most likely to cause the next pandemic. What may improve its transmissibility to the public? A) Increase in consumption of chicken B) Increase in autoimmune disease incidence C) Shift to zoonotic transmission D) Antigenic shift

C) Generating secondary infections with intervals between peaks that approximate the usual incubation period for the infection

9.Person, place, and time characterize the description of an epidemiologic problem. Using knowledge of outbreak investigations, which occurs with propagated outbreaks? A) Same person or vehicle as the reservoir or means of transmission B) Infection transmitted from person to person over a short period of time C) Generating secondary infections with intervals between peaks that approximate the usual incubation period for the infection D) Generating tertiary infections following exposure to a primary case

D) Relationships across two or more generations

9.The community health nurse is using a genogram to aid in the family assessment. What aspect of family connections and relationships is included in the genogram? A) Influence of other systems on families B) Influence of groups on families C) Family relationships and their vital connections D) Relationships across two or more generations

B,C,D

9.The nurse is caring for a client who is from a culture for which the nurse has a limited knowledge base. The nurse conducts the admission interview using cultural humility. Which should the nurse ask in the interview? (Select all that apply.) A)Yes or no questions B)About traditions C)Open-ended questions D)How the client has addressed the illness E)If the cause of the illness is spiritual

B) Ability of a test to correctly identify people who have a health problem or the probability of testing positive if the health problem is truly present

9.Which correctly defines sensitivity? A) The ratio of the incidence rate in the exposed group to the incidence rate in the nonexposed group B) Ability of a test to correctly identify people who have a health problem or the probability of testing positive if the health problem is truly present C) Variations measured in hours, days, weeks, or months and commonly used to quantify outbreaks of infectious disease D) Ability of a test to correctly identify people who do not have a health problem or the probability of testing negative if the health problem is truly absent

D) Actualization or realization of human potential

9.Which is the focus of Smith's eudaimonistic model? A) Elimination of disease or symptoms B) A fit between people and social roles C) Adaptation to the environment D) Actualization or realization of human potential

B) John Graunt

9.Who was one of the first people to study patterns of disease in populations and analyzed the weekly reports of births and deaths in London, the results of these analyses becoming the precursor of modern vital statistics? A) William Farr B) John Graunt C) Florence Nightingale D) John Snow

8. When speaking with an older man in a rural community about his lifelong smoking habit, the nurse is alarmed to learn that he has never heard that cigarette smoking is the major cause of lung cancer. This situation exemplifies which of the seven A's of challenges to elders in rural areas? A) Availability B) Accessibility C) Affordability D) Awareness

Ans: D Feedback: Awareness refers to the level of information dissemination and degree of literacy. Availability refers to the number and diversity of formal services and providers, number of acceptable services, and amount of human service infrastructure. Accessibility refers to the amount of adequate, appropriate, and affordable transportation as well as the degree of cultural and geographic isolation. Affordability refers to the ability to pay for services.

14. A nurse is assessing a client who was recently released from prison. Based on knowledge of the incidence rates of various types of crimes that lead to incarceration, the nurse suspects that the client may have committed which crime? A) Murder B) Rape C) Assault D) Drug possession and trafficking

Ans: D Feedback: Seventy-five percent of all prisoners are nonviolent and have become incarcerated for drug possession and trafficking, bribery, and extortion.

14.The occupational health nurse for a computer manufacturer is required to conduct a physical examination of each new hire. The rationale for this action is to: A) Screen for cardiac defects the worker may be unaware of. B) Ensure the worker's fitness for the job. C) Determine possible causes of the worker's chief complaint. D) Provide a baseline for future comparison. E) Determine the worker's eligibility for the company health insurance plan.

B) Ensure the worker's fitness for the job.

Which exemplify environmental media and transport mechanisms within an exposure pathway? (Select all that apply.) A) Mercury B) Groundwater C) Pond D) Air E) Subsurface soil

B) Groundwater C) Pond D) Air E) Subsurface soil

1.Which of the eight recommended components of a comprehensive school health program should include the physical, emotional, and social conditions that affect the well-being of students and staff? A) Family/community involvement B) Healthy and safe school environment C) Counseling and psychology services D) Health promotion for staff

B) Healthy and safe school environment

17.The occupational health nurse at a company has seen three different employees in one week who have all reported similar upper respiratory symptoms. Which rate should the nurse consider calculating to understand the severity of this problem and to determine whether a cluster or cohort is forming? A) Productivity rate B) Incidence rate C) Prevalence rate D) Ratio of affected to unaffected

B) Incidence rate

19.Which is a community nursing diagnosis? A) Impaired Home Maintenance B) Lack of Health-Seeking Behaviors C) Risk for Imbalanced Nutrition: More than Body Requirements D) Risk for Activity Intolerance

B) Lack of Health-Seeking Behaviors

The nurse travels to Nigeria with a humanitarian aid organization to assess and treat members of a remote community. The children all have high levels of lead in their blood, as a result of exposure to lead dust created by gold mining. Many children are dying. The nurse has never witnessed such a phenomenon in the United States. Which are the most likely reasons for this discrepancy? (Select all that apply.) A) Larger mining operations than in the United States B) Lack of environmental regulations or of their enforcement C) Lack of knowledge concerning environmental dangers D) Weaker immune systems than in the United States E) Need of the miners to earn a living and fewer work options

B) Lack of environmental regulations or of their enforcement C) Lack of knowledge concerning environmental dangers E) Need of the miners to earn a living and fewer work options

18.The community health nurse works in a receiving station after a radiological disaster. The nurse wears the requisite PPE and notes that the majority of the victims have blast injuries. A volunteer worker reports that many of the victims have dust on their clothes. The nurse's response to the dust is to: A)Recognize that the dust is common with blast injuries B)Consider it radioactively contaminated and notify HAZMAT C)Consider the dust a biological agent and therefore a contaminant D)Flush all wounds to prevent chemical contamination

B)Consider it radioactively contaminated and notify HAZMAT

16.Why has there been an increase in the number of acute and chronically ill people residing in the community who need professional nursing care? A)Shorter length of stay in long-term care facilities B)Increase in ambulatory surgery C)Decrease in the use of outpatient clinics D)Increase in length of stay in emergency departments

B)Increase in ambulatory surgery

8.The public health nurse is a first responder in a community disaster, secondary to a flood. One section of the town has experienced a loss of water and electricity for 4 days. The nurse's first action to meet the needs of the community at this time should be to: A)Refer the members of the community to a spiritual outreach program B)Provide access to safe food and water C)Inform the fire department of potential risks within the community setting D)Provide an immunization program

B)Provide access to safe food and water

A nurse working in the rural Southeast United States sees many young white adolescent girls in the clinic. The nurse should be aware that which type of substance use is highest among this demographic? A)Smoking cannabis B)Sniffing glue C)Injecting heroin D)Drinking alcohol

B)Sniffing glue

13.Demographic characteristics indicate that people in developed countries are living longer, healthier lives, yet tremendous health and social disparities exist. Which describes social determinants of health? A)What society does collectively to ensure the conditions exist in which people can be healthy B)Social conditions in which people live and work C)Context of preventing disease and disability and promoting and protecting the health of the entire community D)Comprehensive management of health information and its secure exchange between consumers, providers, government and quality entities, and insurers

B)Social conditions in which people live and work

9. The nurse cares for a client who has symptoms of high fever and unexplained bleeding. After receiving blood test results, the client's primary care provider diagnoses the client with Ebola hemorrhagic fever. What interventions should the nurse anticipate implementing for this client? (Select all that apply.) A)Administration of an antiviral B)Strict infection control C)Isolation of the client D)Rinsing of the client's eyes E)Administration of an antitoxin

B)Strict infection control C)Isolation of the client

14. Why is there an increasing use of home monitoring devices? (Select all that apply.) A)Rising number of people with acute conditions B)Increased need to deliver healthcare to medically underserved populations C)Advances in technology to assess clients' conditions in their home D)Performing assessments in real time using Internet video systems E)Decline in the number of older adults

B,C,D

11.The occupational health nurse for an electronics factory compiles an occupational health history for each worker. Which should be included? (Select all that apply.) A) Location of facility exits and entries B) Chronological record of all past work and potential exposures C) An occupational exposure inventory D) List of exposures in the home or community E) Availability of emergency equipment

B,C,D B) Chronological record of all past work and potential exposures C) An occupational exposure inventory D) List of exposures in the home or community

5.The faith community nurse works in a hospital that serves several local congregations. Which tasks would the nurse likely perform in this role? (Select all that apply.) A) Administer flu shots B) Coordinate the discharge and transition of congregation members from the hospital to home C) Recommend services that the hospital provides D) Serve as a health advisor to members of the congregation E) Assess congregation members for possible upper respiratory illnesses

B,C,D B) Coordinate the discharge and transition of congregation members from the hospital to home C) Recommend services that the hospital provides D) Serve as a health advisor to members of the congregation

21. The Centers for Disease Control and Prevention (CDC) recommends which methods for improving the food environments of school-aged children and adolescents? (Select all that apply.) A) Preserving social norms of adolescents B) School-based prevention C) Policy and regulatory strategies D) Strategic use of media E) Coordination of statewide and local activities

B,C,D,E B) School-based prevention C) Policy and regulatory strategies D) Strategic use of media E) Coordination of statewide and local activities

16. Which health education interventions are appropriate for the school nurse to perform? (Select all that apply.) A) Teaching faculty members how to assess their students for upper respiratory illnesses B) Teaching a student how and when to use her EpiPen C) Teaching a 6th-grade class about the consequences of cyber bullying D) Counseling a student about tobacco use E) Teaching a group of students with eating disorders about a healthy view of food

B,C,D,E B) Teaching a student how and when to use her EpiPen C) Teaching a 6th-grade class about the consequences of cyber bullying D) Counseling a student about tobacco use E) Teaching a group of students with eating disorders about a healthy view of food

21.Which core functions of the government address the health of its citizens? (Select all that apply.) A)Providing healthcare directly to its citizens B)Assessing healthcare problems C)Developing healthcare policy that provides access to services D)Building state-of-the-art hospitals E)Ensuring delivery of healthcare services and achievement of desired outcomes

B,C,E

D)Downstream

9.The nurse researches whether the community would benefit from monthly blood pressure clinics. Which system level would such an intervention address? A)Upstream B)Mainstream C)Sidestream D)Downstream

11. Principles of epidemiology can be used as a method to determine patterns of illness for the faith community, and risk factor assessment helps the nurse decide what programs to provide. Which demographic data about the faith community should the nurse gather for the purpose of assessment? (Select all that apply.) A) Age B) Sex C) Family structure D) Proximity of residence to church E) Proximity of residence to healthcare facilities

A,C,E A) Age C) Family structure E) Proximity of residence to healthcare facilities

4.The Youth Risk Behavior Surveillance Survey (YRBSS) outlines common risk behaviors influencing the health of the nation's youth. Which risk factors are included on this survey? (Select all that apply.) A) Unintentional injury B) Sports-related trauma C) Sleep deprivation D) Sexual behaviors E) Alcohol use

A,D,E

4. Characteristics of a congregation include: (Select all that apply.) A) Organized group of people B) Individuals in the group have diverse religious beliefs, customs, or practices C) External governance structure D) Independent or affiliated with a local or national denomination E) Community within the larger community

A,D,E A) Organized group of people D) Independent or affiliated with a local or national denomination E) Community within the larger community

20.The nurse performs an assessment of a client with schizophrenia. Which comorbid health conditions should the nurse most expect to find in this client? (Select all that apply.) A) Osteoarthritis B) Obesity C) Hypertension D) Breast cancer E) Cardiac complications

B,C,E

B) Genetic changes make the flu virus resistant to the flu shot from year to year.

1. The clinic clients respond negatively to the need for a yearly flu shot. They ask why the medical communities don't just create one shot that will last for several years. Which explanation should the nurse give? A) Genetic changes in the bacteria that cause the flu require a rotating course of antibiotics. B) Genetic changes make the flu virus resistant to the flu shot from year to year. C) There is more than one strain of the flu virus and you must be protected from all of them. D) The shots last only for 3 to 6 months and must be renewed on a yearly basis.

B)Identify a method to improve the health of at-risk clients

1.Community assessment is a critical process for the future because it can: A)Identify the need for a community commission B)Identify a method to improve the health of at-risk clients C)Provide a means of grant funding for clinics D)Provide a client base for at-risk neighborhoods

A,B,C,E

2. The nurse is in charge of a local community program that uses a logic model as a planning and communication tool. Which key components are included in this model? (Select all that apply.) A)Milestones for completing a community health clinic B)Plan to conduct cholesterol screenings C)Need for a nurse practitioner to manage the clinic D)Malaria prevention programs for developing nations E)Possible locations for the health clinic

D) Establish the existence of the outbreak

21. You have heard rumors in the hospital where you work of an Ebola outbreak in your community and you would like to investigate it further. Which of the following should be your first step in the investigation? A) Identify the names of the people infected by the virus B) Formulate and test hypotheses as to the most probable causative factors C) Implement a plan for control of the outbreak D) Establish the existence of the outbreak

A) Fungi C) Bacteria D) Viruses E) Helminths

3. Infectious agents are biological agents capable of producing an infection or infectious disease. Which are considered biological agents? (Select all that apply.) A) Fungi B) Fomites C) Bacteria D) Viruses E) Helminths

C)Community of solution

4. Many in the community have developed cancer. The EPA has discovered a leak of biohazardous waste from a local factory that has leached into the water table. The area is decontaminated and cleaned. The community identifies the need for a local hospice for those with cancer who are dying. Area communities participate in the fund drive. This type of community effort may best be described as: A)Geopolitical community B)Epidemiologic model C)Community of solution D)Developmental model

D) The U.S. Agency for International Development (USAID)

4.Which best exemplifies a bilateral agency? A) The United Nations B) The World Health Organization (WHO) C) The World Bank D) The U.S. Agency for International Development (USAID)

3. Definitions of rural areas are based on which concepts? (Select all that apply.) A) Administrative B) Religious C) Land-use D) Political E) Economic

Ans: A, C, E Feedback: Definitions of rural areas can be based on administrative, land use, or economic concepts, including variations of these three themes. Religious and political concepts are not among the themes on which the definitions of rural areas are based.

7.Public health nursing is distinguished from other specialties by adherence to eight principles. Which are domains of public health nursing practice? (Select all that apply.) A) Intuitive assessment skills B)Community organization skills C)Communication skills D)Cultural competency skills E)Product marketing skills

C)Communication skills D)Cultural competency skills

Which is a relationship risk factor for intimate partner violence? A) Emotional dependence and insecurity B) Belief in strict gender roles C) Desire for power and control D) Unhealthy family interactions

D

1. The client states a history of substance abuse but reports current abstinence. If valid, this report means that the client has not used alcohol or illicit substances in at least how many months? A)3 B)6 C)9 D)12

D) 12

B, E

Which statements regarding the relationship between politics and healthcare are true? (Select all that apply.) A)Politics has a limited effect on healthcare. B)The government should be interested in health matters when a problem affects a specific group or a whole population. C)Politics has only negative effects on healthcare. D)Republicans and Democrats generally agree on healthcare policies. E)Political solutions can be achieved at both the federal and state levels.

A) Incidence density

11.The nurse wishes to measure the number of cases of staph infections that occur in a hospital over the course of 1 year per total client-days during that year. Which measure should the nurse use? A) Incidence density B) Prevalence rate C) Relative risk ratio D) Specific rate

C)Target education and support programs

12. The community health nurse learns that there are statistically high levels of obesity and elevated cholesterol in the community. In addition, the community has a disproportionately high number of fast food restaurants, compared with surrounding communities. Which would be the most appropriate next step, based on these data? A)Gather support in the community to ban fast foods B)Gather further information regarding trans fats C)Target education and support programs D)Create local exercise programs

B,C

16.What services is telehealth used to deliver? (Select all that apply.) A) Chronic care and specialty consultations B) Home telenursing C) Electronic referrals to specialists in expert health facilities D) Transmission of healthcare information to people in the same geographical area

B,D

19.Following a hurricane, the disaster team chooses to use a collaborative model to assess the needs of the community. Each team member is given a community analysis and assigned the task to create a tentative plan of action. Why would the use of a collaborative model hamper the progress of the assessment and plan in a disaster? (Select all that apply.) A)Individual decision-making creates bias. B)Assessment is time-consuming. C)Approach to problem solving is linear. D)Approaches must be preestablished. E)Collaboration is limited.

B,C,D,E

19.The community health nurse can receive a referral to a home health agency at any time. Which phases are included in a home visit? (Select all that apply.) A) Plan personal safety of visit B) Initiating a visit C) Preparation D) Actual visit E) Termination of the visit

A) Measure epidemiologic statistics before and after administering the flu shots. B) Plan and evaluate influenza vaccinations regularly. D) As new data regarding influenza vaccinations become available, modify the intervention accordingly.

19.The nurse is in charge of the flu vaccination program in the community. Which actions should the nurse take when planning and evaluating this program? Select all that apply. A) Measure epidemiologic statistics before and after administering the flu shots. B) Plan and evaluate influenza vaccinations regularly. C) Interview clients regarding their perception of the effectiveness of the vaccinations and use these data in planning future vaccinations. D) As new data regarding influenza vaccinations become available, modify the intervention accordingly. E) Evaluate the success of your program by comparing its results with those in other communities.

B) Aretaeus the Cappadocian

2.Who first described pulmonary tuberculosis in detail? A) Hippocrates of Cos B) Aretaeus the Cappadocian C) Claudius Galen D) Susruta

C) International Committee of the Red Cross

21. A hurricane recently struck a nurse's hometown, and the nurse on behalf of the hospital is coordinating local relief efforts with nongovernmental organizations (NGOs). Which organization should this nurse most expect to work with in this situation? A) Bill and Melinda Gates Foundation B) Living Proof Project C) International Committee of the Red Cross D) Oxfam International

B,C,D

3. Which characterize health inequities? (Select all that apply.) A)Necessary B)Unfair C)Resulting from social injustices D)Avoidable E)Natural

A,B,D,E

5.Which public health entities are often regulated by the U.S. federal government? (Select all that apply.) A) Food B) Medications C) School health D) Devices E) Environment

16.At what age are prodromal assessment symptoms of bipolar disorder typically evident? A) Younger than 3 years of age B) Younger than 6 years of age C) Younger than 9 years of age D) Younger than 12 years of age

A) Younger than 3 years of age

12.Which common problems should a school nurse be prepared to identify in students? (Select all that apply.) A) Vision impairment B) Malignant melanoma C) Attention deficit hyperactivity disorder (ADHD) D) Scoliosis E) Need for orthodontia

A,C,D A) Vision impairment C) Attention deficit hyperactivity disorder (ADHD) D) Scoliosis

9. Supportive relationships with the spiritual head of the congregation are important for the success of a faith community nursing program. Unless the pastor has worked with a faith community nurse before, repeated explanations will most likely be necessary. The spiritual head of the congregation may need further education to clear up the common misconception that the nurse will: A) Provide confidential professional nursing care B) Be a private visiting nurse for the congregation C) Make referrals for appropriate healthcare in the faith community D) Work under the ANA's Faith Community Nursing Scope of Practice

B) Be a private visiting nurse for the congregation

6.Which is an advantage, from the perspective of a faith community nurse, of working in an unpaid congregation-based model? A) Is recognized as a member of the ministry team B) Can limit how much time he or she offers to the congregation C) Receives much assistance from other nurses D) Can serve as an ambassador and referral agent to a health system

B) Can limit how much time he or she offers to the congregation

The nurse conducts biomonitoring of residents of a community exposed to mercury vapor from a manufacturing facility nearby. Which form of biomonitoring is the method for the purpose of determining exposure to this contaminant? A) Measuring heart rate B) Collecting blood or urine C) Assessing respiratory rate D) Obtaining body weight and composition measures

B) Collecting blood or urine

12.The nurse assesses a client who recently attempted suicide by prescription drug overdose. Based on knowledge of a recent study of the findings of the Third National Health and Nutrition Survey, what is the most important risk factor to assess for in this client? A) Substance abuse B) Major depressive disorder C) Social isolation D) Chronic pain

B) Major depressive disorder

20.Which is one of the most important strategies recommended by the Centers for Disease Control and Prevention (CDC) for reducing the spread of HIV? A) Excluding students from school who have symptoms of sexually transmitted infections B) Routinely screening all adolescents and adults aged 13 to 64 C) Requiring parents to take their adolescent children who have sexually transmitted infections for treatment D) Keeping detailed records of all new cases of sexually transmitted infections in people aged 15 to 24 years

B) Routinely screening all adolescents and adults aged 13 to 64

8.The nurse teaches about medication to a client who has just been diagnosed with schizophrenia. The psychiatrist has prescribed clozapine to the client. The primary adverse effect that the nurse should mention is: A) Hypoglycemia B) The metabolic syndrome C) Weight loss D) Dystonia

B) The metabolic syndrome

1. The public health nurse works in New England during the winter months. An ice storm cripples the city, and electrical outages affect the power grid. Trees and frozen power lines are down throughout the city, and the timeline for restoration of power is 7 to 10 days. The management team should classify this occurrence as: A)An emergency B)A natural disaster C)Terrorism D)An accidental disaster

B)A natural disaster

A politically active community health nurse assists in drafting state legislation for alcohol use laws. Which principles of the World Health Organization should be integrated into the draft legislation? (Select all that apply.) A) Legislation should be guided and formulated by the interests of local business owners. B) Alcohol policies should be equitable for all in the state. C) Alcohol policies should be sensitive to cultural contexts. D) Legislation should protect those with substance use disorders from serving extended prison sentences. E )Legislation should protect those exposed to the harmful effects of drinking by others.

B)Alcohol policies should be equitable for all in the state. C)Alcohol policies should be sensitive to cultural contexts. E)Legislation should protect those exposed to the harmful effects of drinking by others.

11.The nurse is locating populations in her region who are exposed to lead-based paint and providing them with information regarding the dangers of lead poisoning. This is an example of which public health intervention? A)Surveillance B)Outreach C)Screening D)Case management

B)Outreach

14.The nursing team cares for victims of a possible anthrax white powder investigation. Which information must be documented by the nurse scribe? (Select all that apply.) A)Victims' names B)Time and place C)Name of the Incident Commander D)General assessment of the field E)Victims' health histories

B)Time and place C)Name of the Incident Commander D)General assessment of the field

12.Select the great public health achievements in the United States in the 20th century. (Select all that apply.) A)Elimination of common infectious diseases B)Recognition of tobacco as a health hazard C)Fluoridation of drinking water D)Healthier mothers and babies E)Decline in the percentage of obese and overweight people

B, C, D

12. Which is an example of a tertiary prevention strategy that a nurse might implement or facilitate for members of a faith community? A) A program on radon detection in the home B) A class on cooking nutritious foods C) An occupational therapy program for patients recovering from strokes D) A water aerobics class for seniors

C) An occupational therapy program for patients recovering from strokes

3.The occupational and environmental health nurse quickly obtains the information needed for accurate assessment and zoom in on the critical aspects of a problem in response to a client situation. The nurse uses highly developed clinical and managerial skills in the work environment. At which American Association of Occupational Health Nurses (AAOHN) competency level in occupational and environmental health nursing is this nurse functioning? A) Advanced beginner B) Competent C) Proficient D) Expert

C) Proficient

19.Who prepared a report for the Massachusetts Sanitary Commission that pointed out that much of the ill health and disability in American cities in 1850 could be traced to unsanitary conditions? A)Clara Barton B)Dorothea Dix C)Lemuel Shattuck D)Lillian Wald

C)Lemuel Shattuck

A woman who is at high risk for alcohol abuse has just found out that she is pregnant. The nurse counsels her about the long-term effects of alcohol on the child. The nurse should include mention of: (Select all that apply.) A)Down syndrome B)Type 1 diabetes C)Low IQ D)Facial anomalies E)Short attention span

C)Low IQ D)Facial anomalies E)Short attention span

The nurse works with a 16-year-old client who has an alcohol use disorder. Which mental disorder, highly correlated to substance use disorder, should the nurse also assess for in this client? A)Obsessive compulsive disorder B)Anxiety disorder C)Major depression D)Bipolar disorder

C)Major depression

4.Which mental illness is most likely to go untreated? A) Anxiety B) Bipolar disorder C) Depression D) Alcoholism

D) Alcoholism

1. In a congregation-based model of nursing, the nurse: A) Serves a health system with assignment to particular congregational settings B) Serves as a liaison between a health system and a congregation C) Contributes to the health and wholeness of people in the context of a faith community D) Serves a particular faith community by virtue of a contract or job description

D) Serves a particular faith community by virtue of a contract or job description

9.Which industry has the highest number of fatal injuries? A) Agriculture B) Construction C) Mining D) Transportation

D) Transportation

B)Implement education in the school setting regarding respiratory and hand hygiene

10. There are 12 students with confirmed cases of the H1N1 flu in grades 6 to 8 at the local middle school in the community. The community nurse advises the caregivers of the ill children to keep the children home for a minimum of 5 to 7 days. Which should be the nurse's next step? A)Recommend closing the school for 6 weeks B)Implement education in the school setting regarding respiratory and hand hygiene C)Require each child in the school to undergo a physical evaluation by the school nurse D)Initiate a system of discipline in the school setting for improper hand washing

9. In researching the problem of homelessness in the city, the nurse discovers that the number of mentally ill homeless has steadily grown over the past couple of decades. The biggest reason for this trend is: A) Deinstitutionalization B) An increase in the prevalence of major mental illness C) Economic downturns D) Shortages of affordable housing

A) Deinstitutionalization

A,C,E

1.Health People 2020 (HP 2020) is a guidepost for nurses and interdisciplinary teams in community and public health. New focal areas for HP 2020 include: (Select all that apply.) A)Adolescent health B)Diabetes C)Genomics D)Nutrition and weight status E)Social determinants of health

C) Air pollution

12.Which risk factor is associated with overall health? A) Allergy B) Traumatic injury C) Air pollution D) Down syndrome

A)Culture

1. A group of teens develop their own method of texting in a language that they feel is all their own. The change is subtle, but the meanings and the feelings associated with the text are known only to a select few. This behavior is an example of: A)Culture B)Cultural competence C)Cultural safety D)Ethnocentrism

D) Self-care Agency

1. The case manager for two families with children who have type 2 diabetes is struck by the differences between the families. In one family, the parents are intentional about providing fruits and vegetables for their child to eat, regularly remind him to check his blood glucose level, and make sure that he engages in some sort of outdoor activity every afternoon. In the other family, the kitchen is stocked primarily with potato chips and processed snack foods, the child does not monitor her glucose level consistently, and she spends her afternoons texting or watching videos on the sofa. By this comparison, the case manager realizes how critical the family is in forming the attitude the child has toward caring for his or her own health. Which grand nursing theory does this experience best illustrate? A) Science of Unitary Beings B) Health as Expanding Consciousness C) Roy Adaptation Model D) Self-care Agency

A) Assisted living facility

1. The nurse discusses home care options with an 85-year-old client who recently fractured her humerus in a fall in her house. She is recovering well now but says the experience has made her see that it is time for her to change her living arrangements. The client can still bathe herself and cook meals on occasion but says she would like to live where nursing care is available onsite and where communal dining and recreation options are offered. Which of the following living arrangements could the nurse recommend? A) Assisted living facility B) Independent living in a senior living facility C) Nursing home D) Apartment located near a hospital

B)Politics

1. The nurse is passionate about research for a cure for pediatric leukemia and has written the state representatives in Congress to urge them to expand federal funding for this research. This behavior is an example of involvement in: A)Policy B)Politics C)Public health D)Equity

C) Agent

1. The nurse works with a female client who recently developed an infection of Staphylococcus aureus while in the hospital. In this case, S. aureus is which component in the chain of infection? A) Host B) Environmental reservoir C) Agent D) Portal

A,C,E

1. Which are recommendations made by the World Health Organization's Commission on Social Determinants of Health (CSDH)? (Select all that apply.) A)Improve conditions under which all people are born, grow, live, work, and age B)Provide increased funding for research of genetic disorders C)Ensure more equitable distribution of power, money, and resources D)Instill a sense of personal responsibility for one's health in each person E)Expand knowledge of the social determinants of health Ans:

A) Rate

1. Which is the primary method used to measure the existence of states of health or illness in a population during a given time period? A) Rate B) Ratio C) Relative risk ratio D) Sensitivity

A) Major cardiovascular-renal disease

1. Which was the leading cause of death in the United States in 1900? A) Major cardiovascular-renal disease B) Influenza and pneumonia C) Tuberculosis D) Gastritis, duodenitis, enteritis, and colitis

A,B,C

11.The nurse decides to use the health belief model for a wellness walking program for older adults. The walk is to begin each day at 8 PM. Each day a different participant is to start a chain of phone calls to other participants to remind all to attend. The nurse informs the clients in the clinic that walking will improve their overall health and well-being. Two older adults show up the first night and none the second. Which information should the nurse gather to explain the failure of the program? (Select all that apply.) A)The severity of the physical challenge related to the client population B)What stands in the way of taking action toward the goal of health promotion C)The presence or absence of cues needed for the program to achieve success D)The presence or absence of a strong team leader among the participants E) The need to increase persuasion when implementing the program

C) High and fluctuating mortality, due to poor health, epidemics, and famine

11.The nurse works to identify epidemiologic transitions in the community. Which exemplifies an epidemiologic transition? A) High fertility and high mortality, resulting in slow population growth B) Improvement in hygiene and nutrition, leading to a decreased burden of infectious disease C) High and fluctuating mortality, due to poor health, epidemics, and famine D) Mortality declines and, later, fertility declines

D) Department of Defense

11.Which agency has an electronic surveillance system for the early notification of community-based epidemics? A) Department of Health and Human Services B) Centers for Disease Control and Prevention C) World Health Organization D) Department of Defense

C,D

11.Who primarily regulates the provisions of home healthcare? (Select all that apply.) A) Insurance companies B) Local government C) State government D) Federal government

B)Preparation

12. The committee has established the need for a sick child clinic in an impoverished neighborhood. The demographic data have been collected, and the committee is ready to meet with a group of local businesspeople who are willing to support the project financially. The plan is for the committee and group to combine to facilitate the establishment of the clinic. Using a transtheoretical model, which level of readiness has the committee achieved? A)Contemplation B)Preparation C)Action D)Relapse

A,B,C

12. The nurse decided to approach local organizations for financial and in-kind support for a new community health initiative. On which local resources can the nurse draw? (Select all that apply.) A)Catholic church B)Branch of Bank of America C)Car dealership D)Community Transformation Grant E)The Bill and Melinda Gates Foundation

A)Degree to which health services increase the likelihood of desired outcomes and are consistent with current knowledge

12. Which most accurately defines quality of care? A)Degree to which health services increase the likelihood of desired outcomes and are consistent with current knowledge B)Improvement of what is wrong or unsatisfactory C)Economic approach or analysis tool used to evaluate the effectiveness of a treatment or intervention D)Presence of a variety of ethnic, racial, and cultural backgrounds of the workers in a specific area such as the health sector

A) Starting an intravenous line for fluid and electrolyte replacement B) Encouraging the client and the client's family to practice good handwashing C) Immediate disinfecting all potentially contaminated objects and surfaces

12.A client arrives on the unit, diagnosed with norovirus infection from eating shellfish. The client has been vomiting repeatedly and is now severely dehydrated. Which interventions are likely to be performed for this client? (Select all that apply.) A) Starting an intravenous line for fluid and electrolyte replacement B) Encouraging the client and the client's family to practice good handwashing C) Immediate disinfecting all potentially contaminated objects and surfaces D) Isolating the client until 12 hours after the client has been symptom free E) Administering a vaccination

B) Use of therapeutic conversation

12.In a 15-minute family interview, after asking a question, the nurse pauses and remains silent while the grandmother, whose English is limited, replies to the question. The nurse makes eye contact with her and nods while she talks. When she is finished, the nurse asks a few follow-up questions. Which principle to maximize information during an interview is demonstrated in this behavior? A) Manners B) Use of therapeutic conversation C) Use of therapeutic questions D) Acknowledgment of family strengths

C) Relative risk ratio

12.Incidence rates for groups exposed to a secondhand smoke are compared with the incidence rates for people who are not exposed to secondhand smoke. This will measure the: A) Incidence density B) Prevalence rate C) Relative risk ratio D) Specific rate

D) "A history of a tick bite is not required."

12.The client is diagnosed with Lyme disease. He states that he is not a hiker and does not remember being bitten by a tick. As part of the education plan, the nurse should state: A) "You must have been bitten by a tick and didn't recognize it as such." B) "Did you spend time in a brushy or grassy area in the last 90 days?" C) "Conduct a full -body tick check once a week during the summer." D) "A history of a tick bite is not required."

C)A subculture

12.The nurse enters a client's home to provide care to a wound and teach the client's wife how to care for the wound. The nurse is comfortable with the client's culture and the fact that it is matriarchal in nature. As teaching begins, the husband interrupts and states that the woman does not change bandages according to his culture. He asks if the nurse is familiar with his culture and then says that all members of his neighborhood follow its principles. The nurse should suspect that the client is a member of: A)A cult B)An orthodox religious group C)A subculture D)An occupation

C) Web of causation

12.The nurse explores all of the possible factors that contribute to coronary artery disease in the community as links in multiple interrelated chains. Which epidemiologic models is the nurse using? A) Epidemiologic triad B) Wheel of causation C) Web of causation D) Natural history

A,B,C,D

12.Which criteria must an individual meet to receive Medicare for home health services? (Select all that apply.) A) Home bound B) Specific plan of care C) Skilled healthcare needs D) Intermittent care needs E) Continuous 24-hour necessity for care

D) Prescription medications

12.Which is the fastest growing healthcare expenditure in the United States? A) Healthcare technology B) Hospital care C) Outpatient treatment and surgery D) Prescription medications

18.A client has been prescribed opioid analgesics. The client asks the nurse how long before the sedation will subside. The nurse should tell the client what time frame? A) 24 to 48 hours B) 48 to72 hours C) 72 to 96 hour D) 1 week

A) 24 to 48 hours

6. According to the U.S. Bureau of Labor Statistics, how many workers in all industries, including state and local governments, had a reportable injury or illness in 2011? A) 3.8 of 100 B) 4.2 of 100 C) 5.8 of 100 D) 6.2 of 100

A) 3.8 of 100

21. The occupational health nurse works in facility located in a region that is highly prone to tornadoes. Which intervention should the nurse make sure is included in the facility's emergency plan to address this potential threat? A) A shelter-in-place policy and procedure B) Personal protective equipment C) Evacuation drills D) Decontamination procedures

A) A shelter-in-place policy and procedure

13. The nurse cares for a client who has a laceration on her arm and reports moderate aching, throbbing pain from the injury. Which medication should the nurse expect to be ordered first for this client? A) Acetaminophen B) Ibuprofen C) Opioids D) Anticonvulsants

A) Acetaminophen

5.Public health nursing is distinguished from other specialties by adherence to eight principles. Which is one of the eight domains of public health nursing practice? A) Analytic assessment skills B) Investigation of disease C) Referral and follow-up D) Case management

A) Analytic assessment skills

Changes to water supplies and increased flooding, which are believed to result from climate change, may make some areas more vulnerable to diseases. Which diseases are expected to become more prevalent as a result of climate change? (Select all that apply.) A) Asthma B) Lung cancer C) Diarrhea D) Malaria E) Dengue fever

A) Asthma C) Diarrhea D) Malaria E) Dengue fever

20.A parish nurse helps an older woman in the congregation who is taking multiple medications develop a system whereby the woman can more easily remember to take the right medications at the right time. Which is the primary ethical principle observed by the nurse in this situation? A) Autonomy B) Confidentiality C) Beneficence D) Nonmaleficence

A) Autonomy

10.The nurse cares for many clients who have serious illnesses. The best candidate for hospice care is the client with A) Brain cancer who is expected to live 3 months and who is not pursuing aggressive treatment B) Congestive heart failure who is expected to live 9 months and who is not pursuing aggressive treatment C) Lung cancer who is expected to live 1 month and who is pursuing radiation therapy D) Stroke who is expected to live only a few more hours unless emergency brain surgery can remove a clot

A) Brain cancer who is expected to live 3 months and who is not pursuing aggressive treatment

20.Who achieved widespread recognition during the Civil War, distributing supplies to wounded soldiers and caring for the casualties with the help of a team of nurses? A)Clara Barton B)Dorothea Dix C)Lemuel Shattuck D)Lillian Wald

A) CLARA BARTON

3.The chart of a client with brain cancer notes "CMO." Which intervention should the nurse most expect this client to receive? A) Chemotherapy for killing the cancer cells B) Radiotherapy for reducing the size of the tumor C) Morphine for pain management D) Antibiotic to fight off an infection

A) Chemotherapy for killing the cancer cells

12.The occupational health nurse for a hydroelectric plant uses root cause analysis to investigate a recent accidental electrocution of a plant worker. The nurse currently is defining the characteristics of the case. In which step of the root cause analysis is the nurse? A) Define the problem B) Collect data C) Identify possible causal factors D) Identify the root cause

A) Define the problem

4.A living will is a document that: A) Describes the client's wishes regarding treatment intended to sustain life B) Designates someone to make medical decisions for the client if he or she is unable to do so C) Permits someone access to the client's finances D) Ensures appropriate end-of-life care

A) Describes the client's wishes regarding treatment intended to sustain life

2. Primary, secondary, and tertiary levels of prevention help reduce risk, identify and limit disabilities, and reduce complications of mental health problems. Which exemplifies a primary prevention strategy for mental health problems? A) Educate families and community groups about mental health issues, symptoms of stress, and barriers to seeking help B) Screen for mental health disorders C) Promote support groups for people with mental health disorders D) Initiate health-promotion activities as a part of rehabilitation services

A) Educate families and community groups about mental health issues, symptoms of stress, and barriers to seeking help

6.A tornado touched down in the city. The public health nurse is a member of the disaster management team. The nurse's role includes: (Select all that apply.) A)Search and rescue B)Clinical assessment and triage C)Provision of health education D)Hazardous materials management E)Protection of property

B)Clinical assessment and triage C)Provision of health education

14. The nurse assesses a 7-year-old girl who has had problems at school lately with her behavior and mood. On the basis of the description provided by the child's mother, the nurse suspects either attention-deficit/hyperactivity disorder (ADHD) or bipolar disorder may be involved. Which behaviors—when occurring in all spheres of the child's life—would most strongly suggest ADHD rather than bipolar disorder in this client? A) Hyperactivity and impulsivity B) Mood and anxiety symptoms C) Irritability and impulsivity D) Anxiety and hyperactivity

A) Hyperactivity and impulsivity

16.The Church Health Center, in developing its curriculum for the preparation of faith-based nurses, identifies seven specific functions that parish nurses perform in faith community work. Spiritual care would occur as part of which function? A) Integrator of faith and health B) Health educator C) Personal health counselor D) Accessing and developing support groups

A) Integrator of faith and health

17.A client has not been able to receive adequate pain control with several different medications that have been administered and is experiencing severe pain. What route of administration might be tried at this time? A) Intraspinal B) Intravenous C) Rectal D) Transdermal

A) Intraspinal

Bioavailability is the amount of the contaminant that: A) Is absorbed into the body and becomes available at the site of physiological activity B) Initially escapes a container and becomes available for contact with living organisms C) Is airborne and available for inhalation D) Is waterborne and available for skin contact

A) Is absorbed into the body and becomes available at the site of physiological activity

Children may be more vulnerable to environmental exposures than adults. There are several factors that increase children's vulnerability. Which factors should be considered with children? (Select all that apply.) A) Their body systems are still rapidly developing. B) They eat less, drink less, and breathe more in proportion to their body size than do adults. C) Their bodies may be less able to break down and excrete contaminants. D) Their behaviors can expose them to more contaminants. E) Their breathing zone is further from the ground.

A) Their body systems are still rapidly developing. C) Their bodies may be less able to break down and excrete contaminants. D) Their behaviors can expose them to more contaminants.

16.As a member of a disaster team, the nurse responds to a confirmed outbreak of the plague. The nurse understands that the pneumonic plague has a high mortality rate. The decision to meet the needs of the public on a wide scale includes: A)Administration of antibiotics via PODs B)Administration of antivirals via PODs C)Door-to-door visits to administer preventative medication D)Isolating designated hospitals to prevent spread of the plague

A)Administration of antibiotics via PODs

During an assessment, a teenager reveals a 2-year history of inhalant use. Using knowledge that fumes may be inhaled through the mouth, this will achieve a state of intoxication similar to: A)Alcohol B)Cannabis C)Cocaine D)Morphine

A)Alcohol

5. Response to a disaster is in its final phase. Although still in the recovery phase of the response, an evaluation meeting is arranged. The purpose of the evaluation meeting is to: A)Analyze the strengths and weaknesses of the response and submit a plan to improve response in the future B)Create a report that emphasizes the cohesiveness of the departmental response to the disaster C)Analyze the effectiveness of the disaster relief teams from respondent states D)Create a postdisaster report for FEMA

A)Analyze the strengths and weaknesses of the response and submit a plan to improve response in the future

4. Five weeks after a category 4 hurricane, a town in southern Louisiana continues to battle for survival. The flood waters have now receded. The nurse is part of the disaster recovery team. Epidemiologic analysis of this town includes factors that influence the health status of this community. Which data may be included in the epidemiologic analysis report? (Select all that apply.) A)Continuing death B)Population shift C)Contamination of water supplies D)Limited access to places of worship E)Collapse of access to healthcare

A)Continuing death B)Population shift C)Contamination of water supplies E)Collapse of access to healthcare

20.The disaster is over and the team prepares to return to normal operations. Before disbandment of the team, it is essential to create a safe environment for the staff. The staff will participate in: A)Critical incident stress debriefing B)Support group to foster recovery C)Information debriefing for prevention purposes D)Interrogation process to identify unsafe behaviors

A)Critical incident stress debriefing

17. The nurse is a member of a disaster team responding to a chemical disaster. The nurse assesses clients in a systematic fashion, concentrating on their ability to breathe. When entering the evacuation site, before the physical assessments, the nurse should: A)Don PPE B)Interview the client C)Report to the hot zone D)Request a HAZMAT expert to witness the client assessment

A)Don PPE

A nurse receives blood test results that indicate that a 14-year-old client has been using cannabis. Based on knowledge of factors that affect rate of cannabis use among youth, which would be the most effective strategy to discourage this behavior? A)Explain the risks associated with cannabis use B)Remind the client that it is still illegal in your state C)Refer the client for counseling D)Share about your own experience with drug use

A)Explain the risks associated with cannabis use

8.Which was a duty performed by district nurses in Liverpool, England, in 1865? A)Use epidemiologic knowledge and methods B)Encourage community organization C)Report facts to and ask questions of physicians D)Assist physicians with surgery in the newly constructed hospitals

C)Report facts to and ask questions of physicians

11. The Children's Defense Fund supports initiatives at the individual, family, community, organizational, and government levels. Encouraging families to spend quality time together by hosting a movie or game night at a local church is an example of a strategy at which level? A) Family B) Community C) Organizational D) Government

Ans: B Feedback: Encouraging families to spend quality time together by hosting a movie or game night at a local church is an example of a strategy at the community level and not at the family, organizational, or government levels.

5. The nurse visits a community health clinic in a remote rural area, at which emergency care is not provided because the facility lacks the medical staff necessary to offer such services. Based on this information, which is the most accurate description of this situation? A) Health professional shortage area B) Medically underserved area C) Medically underserved population D) Undeserved population

Ans: A Feedback: A health professional shortage area (HPSA) is a geographic area, population group, or medical facility with shortages of healthcare professionals to a degree that a full complement of healthcare services is not possible. A medically underserved area (MUA) is an area with a lack of medical care services as determined by the number of primary medical care physicians per 1,000 population, infant mortality rate, percentage of the population with incomes below the poverty level, and percentage of the population 65 years or older. A medically underserved population (MUP) is a U.S. federal designation for those populations that face economic, cultural, or linguistic barriers to accessing primary medical care services. An underserved population is a subgroup of the population that has a higher risk of developing health problems because of marginalization in sociocultural status, access to economic resources, age, or gender.

16. Which accurately defines femicide? A) A homicide of a female that occurs in the context of intimate partner violence B) Violence that targets people or groups of people on the basis of their gender C) A pattern of assaultive and coercive behaviors D) A suicide by a female that occurs in the context of intimate partner violence

Ans: A Feedback: Femicide is a homicide (not a suicide) of a female that occurs in the context of intimate partner violence (IPV). Gender-based violence is violence that targets people or groups of people on the basis of their gender. IPV is a pattern of assaultive and coercive behaviors that is perpetrated against an individual by one who is, was, or wishes to be in an intimate relationship with that individual and that may include inflicted physical injury, psychological abuse, sexual assault, progressive social isolation, deprivation, intimidation, and threats.

21. The nurse cares for a woman who is in her 39th week of gestation and who recently revealed that she is being physically abused by her partner. Which conditions associated with intimate partner violence (IPV) should the nurse look for in this client and in her child now and in the coming weeks? (Select all that apply.) A) Depression in the mother B) Low-birth weight infant C) Down syndrome in the infant D) Gestational diabetes in the mother E) Cleft palate in the infant

Ans: A, B Feedback: Depression before, during, and after birth has been strongly correlated with IPV. Researchers have also found that pregnant women who are assaulted by an intimate partner are at increased risk of giving birth to infants of reduced weight. Down syndrome is a genetic disorder and is not associated with IPV. Gestational diabetes is an endocrine disorder and would not be caused by IPV. A cleft palate is a congenital birth defect not associated with IPV.

12. The nurse suspect a 16-year-old client is being physically abused by her boyfriend. Compared with adolescent girls with no abuse history, this client is at significantly increased risk for: (Select all that apply.) A) Smoking cigarettes B) Anorexia C) Becoming pregnant D) Assaulting her siblings E) Theft

Ans: A, B, C Feedback: Adolescent girls who reported experiencing sexual or physical assault were significantly more likely to report smoking, attempt suicide, use cocaine, become pregnant, and engage in unhealthy eating habits, compared with adolescents with no abuse history. There is no evidence of an association of assaulting one's siblings or theft with exposure to violence.

1. An underserved population is a subgroup of the population that has a higher risk of developing health problems due to marginalization in which areas? (Select all that apply.) A) Age B) Gender C) Sociocultural status D) Political affiliation E) Religion

Ans: A, B, C Feedback: An underserved population is a subgroup of the population that has a higher risk of developing health problems due to marginalization in sociocultural status, access to economic resources, age, or gender. Marginalization in religion and political affiliation are not typically associated with underserved populations.

15. Which instance would be considered child abuse and neglect according to the federal Child Abuse and Prevention and Treatment Act? A) A father leaving a child in the back seat of a car on a hot August day B) A mother not feeding a 5-year-old child all day C) A mother allowing her boyfriend to punch her daughter D) A caregiver smacking away a toddler's hand when she reaches for a hot stove E) A neighbor touching the private parts of a 12-year-old boy

Ans: A, B, C, E Feedback: The federal Child Abuse Prevention and Treatment Act (CAPTA) (42 U.S.C.A. §5106g), as amended by the Keeping Children and Families Safe Act of 2003 (U.S. Department of Health and Human Services, 2003), defines child abuse and neglect as the following: any recent act, or failure to act, on the part of a parent or caretaker that results in death, serious physical or emotional harm, sexual abuse or exploitation, or an act or failure to act that presents an imminent risk of serious harm. A caregiver smacking away a toddler's hand would not result in serious physical or emotional harm. All of the other answers would.

19. Which risk factors are associated with committing gun-related homicide? (Select all that apply.) A) Low income B) Lack of education C) Female gender D) Being a victim of discrimination E) Lack of employment opportunities

Ans: A, B, D, E Feedback: Factors that associated with committing gun-related homicides are as follows: low income, being a victim of discrimination, lack of education, and lack of employment opportunities. Most often, males are the victims as well as the perpetrators of homicides.

6. Which is an accurate health factor or effect of living in rural geopolitical areas? A) Only about 5% of physicians practice in rural America despite that nearly one fourth of the population lives in these areas. B) Although only one third of all motor vehicle accidents occur in rural areas, two thirds of the deaths attributed to these accidents occur on rural roads. C) Rural residents are nearly three times as likely to die from unintentional injuries other than motor vehicle accidents than are urban residents. D) Abuse of alcohol and use of smokeless tobacco is a significant problem among rural older adults.

Ans: B Feedback: Although only one third of all motor vehicle accidents occur in rural areas, two thirds of the deaths attributed to these accidents occur on rural roads. Only about 10% (not 5%) of physicians practice in rural America despite that nearly one fourth of the population lives in these areas. Rural residents are nearly twice (not three times) as likely to die from unintentional injuries other than motor vehicle accidents than are urban residents. Abuse of alcohol and use of smokeless tobacco are a significant problem among rural youth (not older adults).

8. Which statements by a caregiver would indicate the presence of risk factors for violence and should cause the nurse to have significant concern for the safety of children? (Select all that apply.) A) "My boyfriend says that one day he'll put a bullet through my head if I don't stop nagging him." B) "My 3-year-old daughter cries when my partner and I argue." C) "My husband sometimes makes our son come into our bedroom and watch as he hits me." D) "The last time we fought, he threatened to go pull out his shotgun." E) "My husband got upset with how much our daughter was texting and took her phone away from her for a month."

Ans: A, C, D Feedback: Risk factors that indicate significant concern for the safety of the children are as follows: (1) threats to kill the caretaker, children, and/or self or fears on the part of the caretaker for the children's safety; (2) a child was physically injured in an incident in which the caretaker was the target; (3) a child was coerced to participate in or witness the abuse of the caretaker; (4) a weapon was used or threats to use a weapon were made, and the caretaker believed that there was intention and ability to cause harm. A toddler crying because of an argument and a father taking away a daughter's phone for a month are not causes for significant concern regarding the safety of the children.

11. After examining a young woman, the nurse asks her questions about some bruises on her back. The client confides that her husband has started hitting her when they argue. The nurse makes recommendations about getting help; the client dismisses them and states, "It's just a few bruises. I think I can handle it." Which long-term consequences of exposure to violence, according to Healthy People 2020, should the nurse discuss with this client? (Select all that apply.) A) Premature death B) Breast cancer C) Disability D) Lost productivity E) Obesity

Ans: A, C, D Feedback: Healthy People 2020 emphasizes a need to prevent violence and the sequelae known to result from exposure to violence recognizing that violence leads to the following: premature death, disability, poor mental health, high medical costs, and lost productivity. Breast cancer and obesity are not known as long-term consequences of exposure to violence.

6. The nurse is completing a lethality assessment with a female client. Which are appropriate lethality risk questions? (Select all that apply.) A) Is there a gun in your house? B) Are you using drugs or drinking excessively? C) Has your partner ever been arrested? D) What is the worst thing your partner has ever done to you? E) Do you feel that the partner is capable of hurting you?

Ans: A, C, D, E Feedback: Appropriate questions to ask a client to assess for lethality risks associated with intimate partner violence include the following: (1) Is there a gun in the house? (2) Is the woman's partner unemployed? (3) What is the worst thing the partner has ever done to the victim? (4) Does the victim feel that the partner is capable of hurting her, of carrying out his threats? (5) Has the woman's partner ever been arrested? (6) Is the woman's partner (not the woman herself) using drugs or drinking excessively?

21. How can genomics help improve care of underserved populations? (Select all that apply.) A) Identify increased risk in certain populations for specific diseases. B) Boost the immune system of individuals in the community. C) Predict how a community might metabolize medications. D) Reduce the cost of medications. E) Predict a community's unique vulnerability to environmental changes that can control gene expression in some diseases.

Ans: A, C, E Feedback: Understanding the genetic predictors of disease through genomics is a determinant of how certain individuals, families, and communities may be disproportionately affected by illness, and why public health must be part of a strategy that addresses discoveries in a practical way in this developing science. We also know that different groups of people by gender or other key demographic characteristics respond to preventive treatment differently because of their genetic make-up. For example, some genotypes may metabolize medications differently. Some populations may have specific vulnerability and key environmental influences related to their health. Gomes and Pelosi (2013) discuss how the potential vulnerability to environmental changes can control gene expression in diseases of great interest in public health such as cancer, autoimmune diseases, and perhaps even the aging process.

4. Which most accurately defines a medically underserved area? A) A geographic area, population group, or medical facility with shortages of healthcare professionals to the degree that a full complement of healthcare services is not possible B) An area with a lack of medical care services as determined by the number of primary medical care physicians per 1,000 population, infant mortality rate, percentage of the population with incomes below the poverty level, and percentage of the population 65 years or older C) A U.S. federal designation for those populations that face economic, cultural, or linguistic barriers to accessing primary medical care services D) A subgroup of the population that has a higher risk of developing health problems because of marginalization

Ans: B Feedback: A medically underserved area (MUA) is an area with a lack of medical care services as determined by the number of primary medical care physicians per 1,000 population, infant mortality rate, percentage of the population with incomes below the poverty level, and percentage of the population aged 65 or older. A health professional shortage area (HPSA) is a geographic area, population group, or medical facility with shortages of healthcare professionals to a degree that a full complement of healthcare services is not possible. A medically underserved population (MUP) is a U.S. federal designation for those populations that face economic, cultural, or linguistic barriers to accessing primary medical care services. An underserved population is a subgroup of the population that has a higher risk of developing health problems because of marginalization in sociocultural status, access to economic resources, age, or gender.

9. While visiting the home of an 89-year-old woman out in the country, the nurse learns that her primary care provider's office is one hour's drive away. The client states, "I've just stopped going in for appointments in the past few years. It's just too far for me to drive by myself." This situation exemplifies which of the seven A's of challenges to elders in rural areas? A) Availability B) Accessibility C) Affordability D) Awareness

Ans: B Feedback: Accessibility refers to amount of adequate, appropriate, and affordable transportation as well as the degree of cultural and geographic isolation. Availability refers to the number and diversity of formal services and providers, number of acceptable services, and amount of human service infrastructure. Affordability refers to the ability to pay for services. Awareness refers to the level of information dissemination and degree of literacy.

1. Which most accurately defines gender-based violence? A) A homicide of a female that occurs in the context of intimate partner violence B) Violence that targets people or groups of people on the basis of their gender C) A pattern of assaultive and coercive behaviors perpetrated against an individual by one who is, was, or wishes to be in an intimate relationship with the victim D) A type of violence typically committed by one gender but not the other

Ans: B Feedback: Gender-based violence is violence that targets people or groups of people on the basis of their gender. Femicide is the homicide of a female that occurs in the context of intimate partner violence (IPV). IPV is a pattern of assaultive and coercive behaviors perpetrated against an individual by one who is, was, or wishes to be in an intimate relationship with that individual and that may include inflicted physical injury, psychological abuse, sexual assault, progressive social isolation, deprivation, intimidation, and threats. Gender-based violence is based on the gender of the victim, not that of the perpetrator.

18. What percentage of rural children live in poverty? A) 14% B) 24% C) 34% D) 44%

Ans: B Feedback: Nearly 24% of rural children live in poverty.

20. The nurse assesses a new client and performs a standard screening for intimate partner violence. Which is the most appropriate question to ask first? A) "Are you in danger now?" B) "It's normal for couples to disagree about things. What happens when you and your partner disagree?" C) "Do you ever think about hurting yourself, cutting, stopping eating, or purging food?" D) "Have you ever thought about or been granted a restraining order?"

Ans: B Feedback: The most appropriate question to ask first is, "It's normal for couples to disagree about things. What happens when you and your partner disagree?" This question is open-ended and allows the client to share information about behavior that she may not even recognize as abuse. The other questions listed are more appropriate to ask as follow-up questions, once the woman has stated that she is being abused.

13. In the United States, detention centers, jails, and prisons are places that provide safety to the public by incarcerating people who have committed crimes and who are deemed a threat to society. Which demographics are true of the majority of inmates? (Select all that apply.) A) Incarcerated for violent crimes B) Latino or African-American C) Average age of 37 years D) Serving sentences of more than 10 years E) Male gender

Ans: B, C, E Feedback: Most inmates are males who are African-American or Latino. The average age of inmates is 37. Seventy-five percent of all prisoners are nonviolent and have been incarcerated for drug possession and trafficking, bribery, and extortion, and 40% are serving sentences of more than 10 years.

19. Which most accurately characterize Medicare clients with acute myocardial infarctions (AMIs) who are treated in rural hospitals? (Select all that apply.) A) Less likely to experience an opportunistic infection than those in urban hospitals B) Less likely to receive recommended treatments than those in urban hospitals C) More likely to have a second AMI within a month of the first than those in urban hospitals D) Have significantly higher adjusted 30-day post-AMI death rates from all causes than those in urban hospitals E) Experience faster recovery times than those in urban hospitals

Ans: B, D Feedback: Medicare clients with AMI who were treated in rural hospitals were less likely than those treated in urban hospitals to receive recommended treatments and had significantly higher adjusted 30-day post-AMI death rates from all causes than those in urban hospitals. The other answers are not true.

17. Intimate partner violence is a pattern of assaultive and coercive behaviors. This definition includes: (Select all that apply.) A) Emotional injury B) Sexual assault C) Progressive social interaction D) Physical injury E) Deprivation

Ans: B, D, E Feedback: Intimate partner violence (IPV) is a pattern of assaultive and coercive behaviors that is perpetrated against an individual by one who is, was, or wishes to be in an intimate relationship with that individual and that may include inflicted physical injury, psychological abuse, sexual assault, progressive social isolation (not interaction), deprivation, intimidation, and threats. Emotional injury is not included in IPV.

9. A nurse is preparing to make a mandatory report of intimate partner violence (IPV) with regard to the caregiver of a pediatric client. Which actions are appropriate? (Select all that apply.) A) Tell the child about the possibility of filing a mandatory report B) Determine whether it will be safe to inform the child about the report C) Ask about the child's incidence of using drugs D) Ask the victim whether she has a plan to keep herself and the child safe E) Ask the victim if she would like to file a report at the same time as you file yours

Ans: B, D, E Feedback: The nurse should take the following actions when making a mandatory report of IPV: (1) Talk with the adult victim (not the child) about the possibility of filing a mandatory report. (2) Consider the safety concerns of filing. (3) Determine whether it will be safe to inform the children about the report. (4) Share concerns of safety with the Child Protective Agency. (5) Ask about the perpetrator's behaviors with questions such as the following: What is the worst thing he or she has done? Does he or she own a gun? Has he or she been arrested? Does he or she use drugs (not does the child use drugs)? Do you think he or she is capable of hurting you or your children? (6) Address safety planning with the nonoffending victim. (7) Consider filing in concert with the adult victim.

10. The Children's Defense Fund supports initiatives at the individual, family, community, organizational, and government levels. Which exemplifies a strategy at the family level? A) Mentor a child in a READ program. B) Volunteer with children who are homeless or in foster care. C) Create daily homework routines and limit television viewing. D) Vote in every election and advocate for children.

Ans: C Feedback: Individual strategies include mentoring a child in a READ program, volunteering with children who are homeless or in foster care, and voting in every election and advocating for children. A family strategy is to create daily homework routines and limit television viewing.

20. The Children's Defense Fund supports initiatives at the individual, family, community, organizational, and government levels. Which level is represented by hosting a health fair to ensure all children who are eligible for Medicaid or state children's health insurance program are enrolled? A) Family B) Community C) Organization D) Government

Ans: C Feedback: Organizations, not families, communities, or government, are likely host a health fair to ensure all children who are eligible for Medicaid or your state children's health insurance program are enrolled.

4. Preventing intimate partner violence (IPV) and recurrence of violence requires targeting efforts at all three levels of prevention: primary, secondary, and tertiary. Which is a secondary prevention strategy? A) Creating and broadcasting a public service announcement that raises awareness about IPV B) Putting up posters around the health provider's office that give tips on what to do if one is in an abusive relationship C) Screening a woman for evidence of abuse who has several risk factors for IPV D) Providing a woman who is being abused by her husband the number to an abuse hotline so that she can get help leaving him

Ans: C Feedback: Primary prevention includes screening and prevention efforts that target the general population. Secondary prevention occurs through generalized screening and inquiry with high-risk populations. Tertiary prevention involves caring for victims affected by or currently experiencing violence. Creating and broadcasting a public service announcement and putting up posters are examples of primary prevention, as they are aimed at the general public. Screening a woman for evidence of abuse who has several risk factors for IPV is an example of secondary prevention. Providing help to a woman who is being abused is an example of tertiary prevention.

10. Who is in the best position to recognize both children being victimized and children perpetrating violence? A) Pediatric primary care provider B) Parent C) School nurse D) Next door neighboR

Ans: C Feedback: School nurses play a pivotal role in recognizing both children who are being victimized and those who are perpetrating violence, as they are in daily contact with the children. The parents of the children might know whether their own child is a victim or a perpetrator, but they likely would know the other children involved. The child's primary care provider and next door neighbor are not as likely to know of violence experienced or perpetrated by children as the school nurse is, due to the nurse's more frequent contact with the children and location in the school setting.

18. An injury prevention objective of Healthy People 2020 is: A) Prevention of drug-related deaths B) Identification of improper chemical storage in homes C) Decreased incidents of homicide, child maltreatment, and physical assaults D) Elimination of alcohol possession by adolescents on school property

Ans: C Feedback: The injury prevention objectives of Healthy People 2020 include the following: (a) prevention and reduction of firearm-related (not drug-related) deaths; (b) identification of improper firearm (not chemical) storage in homes; (c) surveillance of external causes of injury in emergency departments; (d) decreased incidents of homicide, child maltreatment, and physical assaults; (e) prevention and reduction of sexual assaults; and (f) elimination of weapon (not alcohol) possession by adolescents on school property.

15. The nurse works at a clinic in Arizona that provides bilingual (English and Spanish) care to low-income immigrant workers from Mexico. Which designation most accurately describes the area or the population this nurse serves? A) Health professional shortage area B) Medically underserved area C) Medically underserved population D) Inmates

Ans: C Feedback: A medically underserved population (MUP) is a U.S. federal designation for those populations that face economic, cultural, or linguistic barriers to accessing primary medical care services. A medically underserved area (MUA) is an area with a lack of medical care services as determined by the number of primary medical care physicians per 1,000 population, infant mortality rate, percentage of the population with incomes below the poverty level, and percentage of the population 65 years or older. A health professional shortage area (HPSA) is a geographic area, population group, or medical facility with shortages of healthcare professionals to a degree that a full complement of healthcare services is not possible. An inmate is a person who is held in a jail or prison to protect the public.

17. An older woman in a rural health clinic tells the nurse that this is the first time she's been in a medical facility for more than 10 years. She shows signs of advanced diabetes, including significant necrosis in her feet. When asked why she didn't come in sooner, she says, "I don't drive, myself, and I hate asking my son to bring me—I feel like such a burden." This situation exemplifies which of the seven A's of challenges to elders in rural areas? A) Awareness B) Adequacy C) Acceptability D) Assessment

Ans: C Feedback: Acceptability refers to the degree of reluctance to ask for help. Awareness refers to the level of information dissemination and degree of literacy. Adequacy refers to the level of service standards and evaluation and degree of evidence-based practice. Assessment refers to the amount of basic information available on what is needed using research rigor and analyses.

5. Pediatric providers face special issues in screening for intimate partner violence (IPV) in a caregiver who is accompanying a child. Which are appropriate strategies for addressing this screening issue? (Select all that apply.) A) Asking the mother directly, with the child present B) Asking the child directly, while the mother is out of the room C) Asking the mother indirectly, with a written questionnaire or computer survey D) Screening for the safety and dangerousness of the situation before deciding whether to ask the mother E) Informing parents about mandatory reporting law after asking questions

Ans: C, D Feedback: Appropriate strategies include the following: asking mothers indirectly (written questionnaire, computer survey), asking mothers directly, without (not with) the child present; informing parents about mandatory reporting laws before (not after) asking questions; screening for safety and dangerousness of the situation; and being aware of resources and referrals. It would not be appropriate to ask the child about IPV that the mother may have experienced.

12. The Children's Defense Fund supports initiatives at the individual, family, community, organizational, and government levels. Which strategies are at the government level? (Select all that apply.) A) Provide free tax filing assistance to low-income working families. B) Educate families about how they can apply for food stamps, Head Start, federal nutrition programs, and other similar benefits. C) Ensure children in foster care and detention receive quality treatment to address their mental, behavioral, and emotional needs. D) Promote high-quality children's television programming and access to other quality electronic media. E) Start a halfway house and counseling program for youth who have run away.

Ans: C, D Feedback: Government strategies include ensuring that children in foster care and detention receive quality treatment to address their mental, behavioral, and emotional needs and promoting high-quality children's television programming and access to other quality electronic media. Organization strategies include providing free tax filing assistance to low-income working families and educating families about how they can apply for food stamps, Head Start, federal nutrition programs, and other similar benefits. Community strategies include starting a halfway house and counseling program for youth who have run away.

16. Which qualifies as an urbanized area according to the U.S. Census Bureau? (Select all that apply.) A) A central city with population of 30,000 and surrounding densely settled territory with a population of 25,000 with a combined population density of 500 people per square mile B) A central city with population of 50,000 and surrounding densely settled territory with a population of 10,000 with a combined population density of 750 people per square mile C) A central city with population of 25,000 and surrounding densely settled territory with a population of 25,000 with a combined population density of 1,100 people per square mile D) A central city with population of 20,000 and surrounding densely settled territory with a population of 30,000 with a combined population density of 1,250 people per square mile E) A central city with population of 100,000 and surrounding densely settled territory with a population of 50,000 with a combined population density of 1,500 people per square mile

Ans: C, D, E Feedback: The U.S. Census Bureau defines an urbanized area (UA) by population density. According to this definition, each UA includes a central city and the surrounding densely settled territory that together have a population of 50,000 or more and a population density generally exceeding 1,000 people per square mile.

13. A 50-year-old woman recently underwent a divorce and has two teenaged children. She has an associate's degree from her local community college and earns an annual salary of $22,000. Which risk factors place this woman at risk for intimate partner violence (IPV)? (Select all that apply.) A) Age of 50 years B) Education at associate's degree level C) Recent divorce D) Having two teenaged children E) Annual salary of $22,000

Ans: C, E Feedback: Risk factors include age less than 45 years, low income, lack of employment, recent separation or divorce, education at the high school (not associate's degree) level or less, and having young (not teenaged) children.

14. The nurse helps a client who is a victim of intimate partner violence (IPV) with safety planning. The client worries about what to do if her partner becomes violent again. A safety suggestion for the when the client is at home is for her to go to the: A) Kitchen because it is well-lit B) Bathroom because it is a small space C) Basement because there are places to hide D) Living room because there are windows

Ans: D Feedback: Components for safety planning include a crisis/"disaster" plan, a place to go, how to get there, and other considerations. Safety planning involves discussing measures that the client may not have considered. For example, if an argument occurs at home, the woman should stay away from (1) the kitchen because sharp instruments are there and (2) the bathroom because the space is small and contains many hard surfaces. She should go to a room with a window or door, carry her cell phone, with 911 set to speed dial, and tell family members and neighbors. The basement is not appropriate because it is less likely to have windows or doors.

2. In the Healthy People 2020 federal initiatives in the United States, which group is identified as needing special attention and creative solutions to live a healthy life in the face of sobering health disparities and social injustices? A) Older adults B) People with acute infectious diseases C) People recovering from traumatic accidents D) Mentally ill people

Ans: D Feedback: In the Healthy People 2020 federal initiatives in the United States, the following groups are identified as needing special attention and creative solutions to live a healthy life in the face of sobering health disparities and social injustices: (1) high-risk mothers, (2) chronically ill and disabled people, (3) people living with HIV/AIDS, (4) mentally ill people, (5) substance abusers, (6) homeless people, and (7) immigrants and refugees.

2. Which is a relationship risk factor for intimate partner violence? A) Emotional dependence and insecurity B) Belief in strict gender roles C) Desire for power and control D) Unhealthy family interactions

Ans: D Feedback: Individual risk factors for intimate partner violence include emotional dependence and insecurity, belief in strict gender roles, and desire for power and control. Relationship risk factors include unhealthy family relationships and interactions.

7. Preventing intimate partner violence (IPV) and recurrence of violence requires targeting efforts at all three levels of prevention: primary, secondary, and tertiary. Which is a tertiary prevention strategy? A) Adding a question to the standard health history form that asks whether the client is currently experiencing abuse B) Putting up posters around the health provider's office that give tips on what to do if one is in an abusive relationship C) Screening a woman for evidence of abuse who has several risk factors for IPV D) Providing a woman who is being abused by her husband the number to an abuse hotline so that she can get help leaving him

Ans: D Feedback: Primary prevention includes screening and prevention efforts that target the general population. Secondary prevention occurs through generalized screening and inquiry with high-risk populations. Tertiary prevention involves caring for victims affected by or currently experiencing violence. Adding a question to the standard health history form and putting up posters are examples of primary prevention, as they are aimed at the general public. Screening a woman for evidence of abuse who has several risk factors for IPV is an example of secondary prevention. Providing help to a woman who is being abused is an example of tertiary prevention.

7. Which correctly ranks the top four rural health priorities according to the Healthy People goals? A) (1) Access to quality healthcare, (2) mental health and mental disorders, (3) diabetes, (4) heart disease B) (1) Mental health and mental disorders, (2) access to quality healthcare, (3) heart disease, (4) diabetes C) (1) Diabetes, (2) heart disease, (3) mental health and mental disorders, (4) access to quality healthcare D) (1) Access to quality healthcare, (2) heart disease, (3) diabetes, (4) mental health and mental disorders

Ans: D Feedback: The correct ranking of rural health priorities is (1) access to quality healthcare, (2) heart disease, (3) diabetes, and (4) mental health and mental disorders.

3. Which is a societal risk factor for intimate partner violence? A) Experiencing poor parenting B) Marital instability C) Unwillingness of neighbors to intervene when violence is witnessed D) Traditional gender roles

Ans: D Feedback: Traditional gender roles is a societal risk factor, as it is primarily determined by the society one lives in, rather than by individual characteristics, relationship dynamics, or even local community factors. Experiencing poor parenting is an individual risk factor. Marital instability is a relationship factor. Unwillingness of neighbors to intervene when violence is witnessed is a community factor.

Which most accurately defines gender-based violence? A) A homicide of a female that occurs in the context of intimate partner violence B) Violence that targets people or groups of people on the basis of their gender C) A pattern of assaultive and coercive behaviors perpetrated against an individual by one who is, was, or wishes to be in an intimate relationship with the victim D) A type of violence typically committed by one gender but not the other

B Feedback:Gender-based violence is violence that targets people or groups of people on the basis oftheir gender. Femicide is the homicide of a female that occurs in the context of intimate partner violence (IPV). IPV is a pattern of assaultive and coercive behaviors perpetrated against an individual by one who is, was, or wishes to be in an intimate relationship withthat individual and that may include inflicted physical injury, psychological abuse, sexual assault, progressive social isolation, deprivation, intimidation, and threats. Gender-based violence is based on the gender of the victim, not that of the perpetrator.

A client arrives at the clinic shaky and requesting a refill on a prescription for alprazolam (Xanax). The nurse suspects that the client might be experiencing substance withdrawal. Which comment by the client would most tend to confirm your suspicion? A)"Lately I've had to double up on my dose just for it to be effective." B)"I ran out of pills a week ago and have felt sweaty and agitated ever since." C)"I feel like I just can't function without the medicine." D)"The medicine has been wonderful. I think one more refill should do it for me."

B) "I ran out of pills a week ago and have felt sweaty and agitated ever since."

18.The identification of parental age as a potential risk factor in autism raises the issue that lifetime exposure to environmental toxins might damage the sperm and the ovum. Research findings suggest that the risk of autism in relation to age of the parent at the time of the child's birth is increased by: A) 30% if the father is older than 40 years B) 30% if the mother is older than 35 years C) 40% if the father is older than 35 years D) 40% if the mother is older than 40 years

B) 30% if the mother is older than 35 years

14.Which individual health assessment would a school nurse most likely perform? A) Immunization check B) Assessment of a playground injury C) Vision screening D) Height and weight measurements

B) Assessment of a playground injury

13.Being a school nurse requires which combination of practice type and decision making? A) Collaborative practice and interdependent decision making B) Autonomous practice and independent decision making C) Collaborative practice and dependent decision making D) Autonomous practice and interdependent decision making

B) Autonomous practice and independent decision making

1.The occupational and environmental health nurse faces many different practice issues every day and can function comfortably in roles as a clinician, coordinator, and case manager following company procedures, using assessment checklists and clinical protocols to provide treatment. At which American Association of Occupational Health Nurses (AAOHN) competency level in occupational and environmental health nursing is this nurse functioning? A) Advanced beginner B) Competent C) Proficient D) Expert

B) Competent

16.A client is receiving adjuvant medications to increase the effectiveness to treat the pain associated with end-stage prostate cancer. Which exemplifies an adjuvant medication? A) Opioids B) Corticosteroids C) Antihistamines D) Antivirals

B) Corticosteroids

Environmental justice is an important consideration when working with tribal communities. In which way is traditional tribal practices most often directly affected by contaminated land and water? A) Spiritual practices B) Dietary practices C) Work opportunities D) Exercise opportunities

B) Dietary practices

8.A client has just been pronounced dead. The first action the nurse should take is to: A) Remove all tubes B) Don gloves C) Grasp the eyelashes and gently pull the lids down D) Insert the dentures into the mouth

B) Don gloves

15.The occupational health nurse with a construction company is responsible for filling out a record of accidents and illnesses that occur on the company's work sites. Which source should the nurse consult to acquire the proper record-keeping form and guidelines? A) FMLA B) OSHA C) HIPAA D) NORA

B) OSHA

7.The nurse works with a client who has just been diagnosed with schizophrenia. While discussing the various medication options for this disorder, the nurse explains that some have been associated with the development of type 2 diabetes. The client explains that she is already at risk for developing diabetes and would like to avoid any medications that might promote the disease. On the basis of this information, which medication should this client most avoid? A) Haloperidol B) Olanzapine C) Fluphenazine D) Ziprasidone

B) Olanzapine

6.The school nurse works with a 12-year-old girl who is obese and has type 2 diabetes. As client advocate, which would be most appropriate nursing action? A) Scold the client when she buys a soda from a vending machine at school. B) Schedule a conference with the client's parents to discuss how to collaboratively manage her blood glucose level and encourage her to select appropriate foods in the cafeteria for lunch. C) Perform a finger stick and check the client's blood glucose level when she reports feeling weak and dizzy one day. D) Weigh the client each week in the office to help track her weight and hold her accountable to her weight loss goals.

B) Schedule a conference with the client's parents to discuss how to collaboratively manage her blood glucose level and encourage her to select appropriate foods in the cafeteria for lunch.

The nurse assesses a 15-year-old client who shows some signs of possible drug addiction. The nurse reviews the client's health and family history, looking for key indicators for risk of substance dependence, such as: (Select all that apply.) A)Obesity B)Age at first use of illicit substance C)Family history of alcoholism D)Presence of a learning disability E)Early onset of diabetes

B)Age at first use of illicit substance C)Family history of alcoholism

17.Which organization publishes and articulates the essentials of public health nursing, including the activities and accountabilities that are characteristics of practice at all levels and settings? A)Association of Community Health Nurse Educators (ACHNE) B)American Nurses Association (ANA) C)American Public Health Association (APHA) D)Association of State and Territorial Directors of Nursing (ASTDN)

B)American Nurses Association (ANA)

18.A high school graduate has decided to pursue a career in public health nursing. Which represents the minimum educational credential that this student will need to obtain entry into public health nursing practice? A)Associate's B)Baccalaureate C)Master's D)Doctorate

B)Baccalaureate

15.The nurse is notified that a possible anthrax exposure occurred at the local military base. Civilian workers are routed to the local hospital's emergency department. The nurse's responsibility is to set up PODs in the community. The function of the PODs in response to the anthrax exposure is to: A)Distribute decontaminated food B)Administer prophylactic medications C)Distribute decontaminated water D)Provide triage

B)Administer prophylactic medications

5.Which actions exemplify the school nurse's role as a child advocate? (Select all that apply.) A) Injecting a child with her EpiPen during an anaphylactic reaction to peanuts B) Convincing the cafeteria manager to include low-sugar options in the lunch menu for students with diabetes C) Taking the temperature of a child who is exhibiting signs of an upper respiratory infection D) Teaching a child who has recently been diagnosed with asthma how to use an inhaler E) Instructing a group of faculty members on the proper way to respond to a child having an epileptic seizure in class

B,D,E B) Convincing the cafeteria manager to include low-sugar options in the lunch menu for students with diabetes D) Teaching a child who has recently been diagnosed with asthma how to use an inhaler E) Instructing a group of faculty members on the proper way to respond to a child having an epileptic seizure in class

14.Which are examples of primary prevention activities? (Select all that apply.) A) A program helping military veterans with amputations adapt to their new circumstances B) HIV screening open to the whole faith community C) A weight-lifting program for seniors to maintain strong bones D) A community vegetable garden to promote healthy eating E) Leg exercises to help people recovering from hip fractures to walk again

C,D C) A weight-lifting program for seniors to maintain strong bones D) A community vegetable garden to promote healthy eating

21.The faith community nurse recently visited with a member of the congregation, Sandy, who is recovering from a hysterectomy. Following a church service one Sunday, another member of the congregation, Patricia, approaches you and says, "How is Sandy recovering from her surgery?" Which is the most appropriate response? A) "She still has some bleeding, but overall she's doing well." B) "You mean her hysterectomy? She's doing really well." C) "I'm afraid I can't share any details, but I'm sure she'd love a call or text from you." D) "I'm sorry, Patricia, but that's none of your business."

C) "I'm afraid I can't share any details, but I'm sure she'd love a call or text from you."

15. A client has been prescribed a nonopioid analgesic for mild-to-moderate pain. The client has normal liver function. What is the maximum recommended amount of acetaminophen for this client to take per day to avoid liver damage? A) 2 g B) 3 g C) 4 g D) 5 g

C) 4 g

In the United States, most citizens have access to clean water and sanitation services and often take these services for granted. Worldwide, however, clean water and sanitation are not standard. How many people worldwide in 2010 lacked access to improved water sources? A) 7 million B) 78 million C) 783 million D) 7.8 billion

C) 783 million

20.A dying client with inadequate fluid intake is experiencing constipation. What would be an appropriate nursing intervention for this client? A) Administer psyllium B) Tap water enema C) Administer lactulose D) Apricot juice

C) Administer lactulose

11.Which occurred as a result of the passage of the Education for All Handicapped Children Act in 1975? A) Children with disabilities were provided tax-funded tutors for home education. B) Children with mental disabilities were entitled to a tax-funded education at a special education private school. C) Children with chronic or complex medical issues were entitled to a public education. D) Children with physical disabilities were provided grants to attend the college of their choice.

C) Children with chronic or complex medical issues were entitled to a public education.

3.The nurse assesses a client who recently immigrated to the United States from Haiti. The client reports upset stomach and diarrhea. He explains that a witch doctor put a curse on him and that he sometimes sees evil spirits around him. The most accurate way to describe this type of illness would be as a: A) Mental illness B) Gastrointestinal disorder C) Culture-bound syndrome D) Hypochondria

C) Culture-bound syndrome

17.In which phase of a school-based substance abuse program is information most likely to have meaning and applicability to students? A) Inoculation B) Primary C) Early relevance D) Tertiary

C) Early relevance

18.The occupational health nurse for a multinational corporation compares aggregate data on the occurrence of lung cancer in workers in a factory in Brazil with workers in a factory in Hong Kong. Which type of epidemiologic study is this nurse most likely conducting? A) Prospective, cohort B) Case-control C) Ecological D) Cohort case-control

C) Ecological

4.Which exemplifies a physical hazard? A) Infectious agents B) Hazardous drug and toxin exposures C) Electric and magnetic fields D) Sexual harassment

C) Electric and magnetic fields

15. The Church Health Center, in developing its curriculum for the preparation of faith community nurses, identifies seven specific functions that parish nurses perform in faith community work. Which exemplifies the health advocate function of the faith community nurse? A) Individual health teaching B) Group health teaching C) Empowerment of members of the congregation D) Provider of health resources

C) Empowerment of members of the congregation

Several important pieces of legislation have helped highlight the importance of environmental health and environmental protection. Which best describes Superfund? A) National program to control the damaging effects of air pollution B) Program that protects and enhances the quality of the nation's air by regulating stationary and mobile sources of air emissions C) Environmental program established to address abandoned hazardous waste sites D) Comprehensive framework of standards, technical tools, and financial assistance to address the many causes of pollution and poor water quality

C) Environmental program established to address abandoned hazardous waste sites

10.The history of school nursing can be traced back to 1902, when Lillian Wald was working in a New York City public school. She viewed school nursing services as a way to decrease which problem? A) Nutrition deficiencies B) Dental cavities C) Excessive absenteeism D) Seasonal allergies

C) Excessive absenteeism

6. Public health nursing is distinguished from other specialties by adherence to eight principles. Which is one of the eight domains of public health nursing practice? A) Policy development and individual planning skills B) Individual dimensions of practice skills C) Financial planning and management skills D) Leadership and individual critical thinking skills

C) Financial planning and management skills

Which most accurately describes environmental epidemiology? A) Focuses on the amount of a contaminant that is absorbed into the body B) Involves monitoring the results of medical tests to determine whether a person has been exposed to a contaminant C) Focuses on the incidence and prevalence of disease or illness in a population from exposures in their environments D) Involves monitoring the number of adverse effects of chemical, physical, or biological agents on people, animals, and the environment

C) Focuses on the incidence and prevalence of disease or illness in a population from exposures in their environments

12.After a hurricane, the nurse is placed on a planning committee to meet the direct needs of the community. Most of the clean-up and rescue work has been completed, and several members of the community are living in shelters and are homeless. One of the foremost priorities of the committees is to establish: A)Ecumenical centers for long-term worship B)Home repair resources C)POD emergency supply centers D)Financial aid centers

C)POD emergency supply centers

2.The national weather service warns of an impending ice storm that may have crippling effects on the community. Widespread electrical outages are predicted as well as freezing temperatures. The community health nurses are mobilized to establish emergency living quarters and clinics for those without heat. The nurses' actions are occurring during which phase of the disaster? A)Impact B)Postimpact C)Preimpact D)Preparedness

C)Preimpact

3. In a parish nursing model, the nurse: A) Serves a health system with assignment to particular congregational settings B) Serves as a liaison between a health system and a congregation C) Is part of the ministry staff of the congregation D) Serves a particular faith community by virtue of a contract or job description

C) Is part of the ministry staff of the congregation

Which is the most accurate definition of an exposure pathway? A) The total amount of a contaminant that comes in direct contact with the body B) Factor that determines a person's level of exposure to a contaminant C) Method by which people are exposed to an environmental contaminant D) Process to determine whether exposure to an environmental contaminant has occurred

C) Method by which people are exposed to an environmental contaminant

3. A nurse successfully persuades an obese client to perform a weekly weigh-in at home using a digital scale and record the weight in a log. This strategy is an example of: A) Telehealth B) Health information technology C) Personal responsibility for health D) Evidence-based nursing

C) Personal responsibility for health

7.The nurse is setting up a faith community nursing program at a local synagogue and would like a better understanding of how the overall congregation is structured and organized. Who would be best to ask about the structure of the congregation? (Select all that apply.) A) Lay members B) Physician in the congregation C) Rabbi D) Health system administrator

C) Rabbi

7.The nurse cares for a client who is near death. For which change, indicating that death is imminent, should the nurse observe? A) Deep breaths with periods of apnea B) Mottling of upper extremities C) Skin cool and dusky D) Signs of respiratory alkalosis

C) Skin cool and dusky

17.Which is an example of a family assessment? A) Blood pressure screening B) Home assessment for safety C) Stress assessment for the family with a new baby D) Assessment for normal grieving for the recently widowed

C) Stress assessment for the family with a new baby

13.The nurse is speaking with a 14-year-old client who has attempted suicide by cutting his wrists in the past. How would the nurse implement a means-restriction approach to suicide prevention in this case? A) Teach the client to block out all thoughts of self-harm as soon as they arise B) Encourage the client to seek out counseling C) Suggest to the client's parents that they lock up all sharp knives in the house D) Recommend to the client's parents that the client be under constant surveillance

C) Suggest to the client's parents that they lock up all sharp knives in the house

19.The nurse is appointed to be a member of the risk communication team during a disaster response situation. During an evacuation, the nurse is informed that an increasing number of individuals are reluctant to leave their homes. To complete an orderly evacuation, it is necessary to: A)Involve the armed forces B)Declare martial law C)Create an environment of trust and credibility D)Disseminate information regarding the necessity of the evacuation

C)Create an environment of trust and credibility

9.Which is a Healthy People 2020 goal? A)Decrease the incidence of preventable disease, disability, injury, and premature death B)Reduce health inequity, decrease disparities, and improve the health of certain groups C)Create social and physical environments that promote good health for all D)Promote healthy development and healthy behaviors for the geriatric stage of life

C)Create social and physical environments that promote good health for all

The nurse works with a recovering heroin addict who is currently receiving opioid substitution treatment. Besides reducing his use of heroin, what additional benefits are offered by this treatment? (Select all that apply.) A)Reduction in alcohol use B)Weight loss C)Decrease in transmission of HIV D)Improving the client's overall health E)Decrease in depression

C)Decrease in transmission of HIV D)Improving the client's overall health

The nurse teaches a client about pharmacologic treatments for alcohol-dependent disorders. The client correctly understands the teaching when she tells the nurse that a medication that induces aversive effects if alcohol is consumed is: A)Naltrexone B)Acamprosate C)Disulfiram D)Buprenorphine

C)Disulfiram

The community health nurse is concerned with the prevalence of new HIV infections in the community and understands the strong correlation between substance use disorders and HIV. Which interventions should the nurse promote in the community health clinic to best address this problem? (Select all that apply.) A)12-step self-help groups B)Family and couples therapy C)Opioid substitution treatment D)Syringe exchange programs E)Motivational interviewing

C)Opioid substitution treatment D)Syringe exchange programs

5.On the basis of the understanding of demographic risk factors related to mental illness, which individual would be most likely to have an untreated psychiatric disorder? A) 50-year-old Caucasian man B) 35-year-old Korean woman C) 12-year-old Hispanic girl D) 18-year-old African-American man

D) 18-year-old African-American man

18.Which level of assessment occurs when assessing whether healthcare is accessible to members of the congregation? A) Individual B) Family C) Congregation D) Community

D) Community

18.Which is the most appropriate intervention to use in a school-based substance abuse program? A) Integrating students' goals with those of a prevention program B) Assessing the needs and interests of the group carefully after selecting a program C) Designing a program that is knowledge based D) Designing an interactive, behavior-focused program

D) Designing an interactive, behavior-focused program

Which is the most accurate definition of a risk assessment? A) Assessment of the adverse effects of chemical, physical, or biological agents on people, animals, and the environment B) Process to help determine whether an individual has been exposed to environmental contaminants C) Assessment of factors that determine a person's level of exposure to an environmental contaminant D) Determination of the likelihood of adverse effects in a group exposed to an environmental contaminant

D) Determination of the likelihood of adverse effects in a group exposed to an environmental contaminant

11.There is substantial variation in the incidence of mood disorders and suicide across cultures worldwide. The two most consistent factors associated with the incidence of depression and anxiety are: A) Number of parents in the home and education level B) Degree of social support and employment status C) Number of parents in the home and degree of social support D) Education level and employment status

D) Education level and employment status

2.The occupational and environmental health nurse provides leadership in developing occupational safety and health policy within the organization and function in an upper management role. The nurse has served as a consultant to both business and government and has designed and conducted significant research. At which American Association of Occupational Health Nurses (AAOHN) competency level in occupational and environmental health nursing is this nurse functioning? A) Advanced beginner B) Competent C) Proficient D) Expert

D) Expert

4.A nurse performs a variety of tasks as part of the nurse's position at a hospital. Which task best exemplifies public health? A) Reading current nursing journals and integrating the latest research into daily practice B) Instructing a client on how to best care for a suture site at home C) Participating in a videoconference call with a client who lives in a remote area D) Facilitating a community-wide smoking cessation program one month out of the year

D) Facilitating a community-wide smoking cessation program one month out of the year

2. The leading cause of death in 2010 was: A) Cerebrovascular disease B) Chronic lower respiratory disease C) Diabetes mellitus D) Heart disease

D) Heart disease

14. The nurse cares for a hospice client who reports pain from nerve damage related to diabetic neuropathy. She rates the pain at 9 out of 10. Which medication should she expect the physician to order for this client? A) Antidepressant B) Muscle relaxant C) Low doses of opioid D) High doses of opioid

D) High doses of opioid

10.According to current research findings, which group has the most experience with congregational health ministers? A) Mainline Protestant and African-American Protestant B) Conservative Protestant and Roman Catholic C) African-American Protestant and Conservative Protestant D) Mainline Protestant and Roman Catholic

D) Mainline Protestant and Roman Catholic

15.The school nurse is concerned about the lack of nutritional choices available in the food vending machines at the school. Which is the best tool to address this situation? A) Food-Safe Schools Action Guide B) Health Education Curriculum Analysis Tool C) Improving the Health of Adolescents and Young Adults: A Guide for States and Communities D) Making It Happen!

D) Making It Happen!

Environmental justice is most accurately described as the belief that: A) Those who violate the environment should be required to pay significant fines or face extensive prison sentences. B) All the harms humans have committed against the environment will one day result in divine judgment. C) All people should have equal access to the environment. D) No group should suffer more from environmental health consequences than others.

D) No group should suffer more from environmental health consequences than others.

19.A Muslim client is receiving end-of-life care. As death approaches, what special considerations should the nurse take in the care of this client? A) Speak with the family about arrangements for cremation B) Schedule a visit from the chaplain for special rites C) Arrange for grief counseling for the client D) Place the client in a special position

D) Place the client in a special position

1.Primary, secondary, and tertiary levels of prevention help reduce risk, identify and limit disabilities, and reduce complications of mental health problems. Which exemplifies a tertiary prevention strategy for mental health problems? A) Screen for mental health disorders B) Refer high-risk people for diagnostic services C) Provide mental health services following stressful community events D) Promote support groups for people with mental health disorders

D) Promote support groups for people with mental health disorders

There are different methods for conducting risk assessment for environmental contaminants. Which formula determines the amount of risk? A) Hazard = exposure × risk B) Exposure = risk × hazard C) Risk = contaminant presence × exposure D) Risk = hazard × exposure

D) Risk = hazard × exposure

21.The nurse works with a client who was just diagnosed with major depression. Which would the nurse most suspect will be prescribed as the first line of treatment for this client? A) Tricyclic antidepressant (TCA) B) Electroconvulsive therapy (ECT) C) Stimulant D) Selective serotonin reuptake inhibitor (SSRI)

D) Selective serotonin reuptake inhibitor (SSRI)

10.Epidemiologic studies are essential to target programs to populations in greatest need. To help measure progress toward improving mental health, which is a primary focus of epidemiologic studies? A) Physical costs B) Emotional costs C) Mental costs D) Social costs

D) Social costs

17.The school nurse follows up with a third grader who was recently diagnosed with attention deficit/hyperactivity disorder (ADHD) to make sure that he takes his prescribed medication. What type of medication would the nurse most suspect this client is taking? A) First-generation antipsychotic B) Second-generation antipsychotic C) Mood stabilizer D) Stimulant

D) Stimulant

10.A nurse is coordinating a plan to bring vaccinations in a cost-effective way to a rural community that currently lacks access to them. This is an example of which public health intervention? A)Surveillance B)Outreach C)Screening D)Case management

D)Case management

The nurse counsels a college student who admits to frequent binge drinking. Based on her age and environment, the most likely reason for this student's binge drinking is to: A)Deal with family dysfunction B)Cope with violence C)Escape painful emotions D)Enhance mood or affective state

D)Enhance mood or affective state

The nurse works with a client who is struggling with heroin addiction. Which examination should the nurse perform to screen for a comorbidity that is highly associated with substance use disorders? A)Assessment of blood glucose level B)Bone marrow aspiration C)Magnetic resonance imaging of the brain D)Mental status examination

D)Mental status examination

The nurse works in the ER when a client arrives with respiratory depression associated with a heroin overdose. Which medication should the nurse expect to be ordered for this client? A)Naltrexone B)Acamprosate C)Disulfiram D)Naloxone

D)Naloxone

11. The nurse is responsible for triage in a setting where a natural disaster has occurred. The triage is expected to last for an extended period of time. Which triage model would be most effective in this scenario? A)Simple triage and rapid treatment (START) B)Prehospital triage C)JumpSTART triage D)Start/Save triage

D)Start/Save triage


Kaugnay na mga set ng pag-aaral

Maternal & Child Practice Exam 1

View Set

General Anthropology Final Exam Study Set

View Set

LARA-NOUR'S ULTIMATE 2020 AP ART HISTORY STUDY GUIDE

View Set

Biochemistry Exam 3 Study Guide Questions

View Set